Die Allgemeine Relativitätstheorie ist ja bekanntlich schwer zu verstehen – ein Grund, warum es hier soviele Artikel dazu gibt. Aber eine – wenn nicht sogar die – Grundidee dahinter ist eigentlich ziemlich simpel, denn letztlich hat Einstein nichts gemacht, als die beiden Newtonschen Gesetze zu vertauschen.

Betrachten wir ein klassisches Beispiel (auch in diesem Video zu finden): Ein Apfel hängt an einem Baum, löst sich irgendwann und fällt dann herunter – der Legende nach genau auf Newtons Kopf oder Nase.

Newton würde das so beschreiben: Solange der Apfel am Baum hängt, wirkt insgesamt keine Kraft auf den Apfel, denn ein Objekt, auf das keine Kraft wirkt, bewegt sich mit konstanter Geschwindigkeit (in diesem Fall mit Geschwindigkeit Null, weil der Apfel ja seinen Ort nicht ändert). Das ist das berühmte erste Newtonsche Gesetz:

Wenn keine Kraft wirkt (oder sich alle Kräfte aufheben), bewegt sich ein Objekt gleichförmig mit konstanter Geschwindigkeit auf einer geraden Linie.

newtonApfel1a

Wenn man genau hinguckt, dann sieht man, dass eigentlich zwei Kräfte auf den Apfel wirken – die Schwerkraft zieht den Apfel nach unten, aber der Ast zieht ihn mit gleicher Kraft nach oben, was man auch daran sehen kann, dass der Stängel selbst eine Kraft erfährt – mit einem hinreichend genauen Messaufbau könnten wir sehen, dass er ein wenig elastisch gedehnt wird. Die beiden Kräfte sind gleich groß, aber genau entgegengesetzt zueinander, deshalb heben sie sich auf.

Und wenn der Stängel dann irgendwann zu schwach geworden ist, um den Apfel zu halten, dann löst sich der Apfel. Jetzt überträgt der Stängel keine Kraft mehr, es wirkt nur noch die Schwerkraft, und die beschleunigt den Apfel nach unten, so dass er zu Boden fällt. Hier schlägt also das zweite Newtonsche Gesetz zu:

Kraft ist Masse mal Beschleunigung. (folgt dem Link für eine ausführliche Erklärung des ersten und zweiten Newton-Gesetzes)

newtonApfel1b

Graphisch kann man das, was passiert, in einem kleinen Diagramm veranschaulichen, in dem man auf der horizontalen Achse die Zeit und auf der senkrechten Achse den Ort aufträgt. So sieht das Diagramm dann aus (Idee dazu aus dem oben verlinkten Video):

newtonRaumzeit1

Am Anfang ist der Apfel in Ruhe, im Diagramm ist sein Weg also eine gerade und horizontale Linie, der Ort ändert sich ja nicht. Wenn der Apfel fällt, ändert er seinen Ort mit der Zeit und wird dabei immer schneller, also wird die Linie gekrümmt. (Eine schräg verlaufende gerade Linie würde bedeuten, dass der Apfel sich mit konstanter Geschwindigkeit bewegt, dann wäre er auch kräftefrei.)

Soweit Newton. Aber Einstein hatte einen Einwand dagegen: Normalerweise merken wir es, wenn Kräfte auf uns wirken – wenn wir im Auto sitzen und das Auto beschleunigt, dann werden wir in die Sitze gedrückt, wenn ein Fahrstuhl uns nach oben beschleunigt, dann werden wir nach unten gedrückt. Wenn wir dagegen im freien Fall sind, dann merken wir davon nichts – ein frei fallendes Objekt ist schwerelos, so wie die Astronautinnen in einer Raumstation. (Die sind auch nicht schwerelos, weil die Station so weit von der Erde weg ist…) Einstein überlegte eine Weile herum, ob es irgendeine Möglichkeit gibt, zu merken, dass man gerade im freien Fall ist, und stellte schließlich fest, dass es diese Möglichkeit lokal nicht gibt. “Lokal” bedeutet, dass ihr nur Experimente machen dürft, die eine kurze Zeit dauern und auf einen kleinen Raumbereich beschränkt sind.  (Natürlich gilt es nicht, einfach zu gucken, ob sich der Boden nähert, und wenn man weiter entfernte Objekte hat, die auch fallen, dann merkt man, dass die sich anders bewegen – zwei frei fallende Steine zum Beispiel bewegen sich ja ein wenig aufeinander zu, weil sie beide zum Erdmittelpunkt fliegen).

Einstein hatte ja die Angewohnheit, einfache Ideen zum Prinzip zu erklären und zu sehen, wo man damit landet (“Hey, was wäre, wenn die Lichtgeschwindigkeit für alle Beobachter immer gleich wäre…”). Wo landet man also, wenn man diese Erkenntnis zu Ende denkt?

Da wir es normalerweise eben nicht merken, wenn Kräfte auf uns wirken und da wir eine Bewegung im freien Fall (lokal) in keiner Weise von einer Situation unterscheiden können, in der wir irgendwo fernab jeder Schwerkraft oder sonstiger Einflüsse im Weltall schweben, sagte Einstein: O.k., wenn wir im freien Fall sind, dann wirken also keine Kräfte auf uns. Den Apfel, der gerade vom Baum fällt, beschreiben wir also korrekterweise nicht mit dem zweiten Newtonschen Gesetz (Kraft ist Masse mal Beschleunigung), sondern mit dem ersten Gesetz, denn er ist kräftefrei.

“Äh, ‘tschuldigung, Albert, aber ist dir schon mal aufgefallen, dass ein frei fallendes Objekt seine Geschwindigkeit ändert? Sowas nennt man doch wohl eine Beschleunigung, oder nicht?” Ja, dieser Einwand liegt nahe und ist berechtigt.

Aber wir können trotzdem an Einsteins Idee festhalten – dazu müssen wir “nur” neu definieren, was es bedeutet, sich gleichförmig zu bewegen. Unter anderem deshalb hat Einstein das Konzept der gekrümmten Raumzeit eingeführt (und wenn ihr rechts bei der Tag-Wolke klickt oder die Suchmaske bemüht, findet ihr einen Haufen Artikel zum Thema) – in der gekrümmten Raumzeit ist der “geradeste und gleichförmigste” Weg der, bei dem der Apfel zur Erde fällt. (Vornehm sagt man “Der Apfel folgt einer Geodäte”.)

Der frei fallende Apfel folgt der geradesten Bahn in der gekrümmten Raumzeit, die es für ihn gibt – und genau deswegen merkt der Apfel auch nichts von irgendwelcher “Schwerkraft”. (Relativ ausführlich habe ich gerade in einer kleinen Artikelserie erklärt, wieso aus der Einsteinschen Theorie dasselbe rauskommt wie bei Newton.)  Für ihn gilt also jetzt das erste Newtonsche Gesetz.

einsteinApfel1b

Wenn der Apfel dagegen still und ortsfest am Baum hängt, dann folgt er eben nicht der geradesten und gleichförmigsten Bahn, die es für ihn gibt – dann müsste er nämlich runterfallen. Der Stängel und der Ast üben eine Kraft auf ihn aus, und jetzt wird er beschleunigt, nämlich von der Bahn weg, die er sonst eigentlich nehmen würde. Für uns sieht es so aus, als ob der Apfel ortsfest und ohne jede Beschleunigung am Ast hängt – aber das liegt nur daran, dass wir auch die ganze Zeit beschleunigt werden, weil der Boden unter unseren Füßen uns daran hindert, unserer geradest-möglichen Bahn zu folgen. Der ruhig hängende Apfel ist also der, der beschleunigt wird, für ihn gilt das zweite Newtonsche Gesetz, obwohl er ortsfest ist.

einsteinApfel1a

Auch die Beschreibung nach Einstein kann man in einem Diagramm einzeichnen – das sieht etwas ungewohnt aus, weil die Diagrammachsen gekrümmt sein müssen, um die gekrümmte Raumzeit zu symbolisieren:

einsteinRaumzeit1

Hier ist die grüne Linie, die den Weg des Apfels darstellt, am Anfang gekrümmt, um der gekrümmten Raumzeit zu folgen, sobald der Apfel fällt, ist die Linie gerade. Der Weg des Apfels ist aber bei Einstein und Newton (fast) genau derselbe – messbare Unterschiede gibt es zwischen beiden Theorien nur, wenn das Schwerefeld bzw. die Raumzeitkrümmung groß ist (wie in der Nähe der Sonne) oder wenn wir die zeitliche Änderung von Gravitationsfeldern betrachten (wie bei Gravitationswellen).

Ausgehend von dieser Grundidee hat Einstein schließlich die Allgemeine Relativitätstheorie entwickelt – die enthält natürlich noch ein klein wenig mehr als nur diese veränderte Betrachtungsweise. Aber die Grundidee ist die, die ich hier skizziert habe und die Einstein als den “glücklichsten Gedanken meines Lebens” bezeichnete: Ein frei fallender Apfel bewegt sich nach dem ersten Newtonschen Gesetz, ein ruhig am Baum hängender Apfel nach dem zweiten, und nicht andersherum.

PS: Damit keine Missverständnisse aufkommen: Das Vertauschen der Newtongesetze gilt natürlich nur für die Schwerkraft: Wenn ihr in eurem Auto beschleunigt, dann gilt auch nach Einstein das zweite Newton-Gesetz mit F=ma – diese Beschleunigung hat  nichts mit der Krümmung der Raumzeit zu tun.

Kommentare (103)

  1. #1 KarlH
    28. August 2016

    Fehlt nur der entscheidende Hinweis darauf, dass diese Betrachtung nur deswegen möglich ist, weil die schwere und träge Masse eines Körpers äquivalent sind. Das heißt, dass Körper mit verschiedener Masse auf Grund der Schwerkraft gleich schnell fallen, genauer gesagt, sogar unter identischen Anfangsbedingungen den gleichen raumzeitlichen Bahnverlauf haben.

  2. #2 MartinB
    28. August 2016

    @KarlH
    Ja, ich habe überlegt, ob ich das noch dazu schreiben soll, habe mich dann aber dagegen entschieden, weil ich den Artikel auf den Kerngedanken reduzieren wollte. Das Äquivalenzprinzip kann man ja auch umgekehrt als die direkte Folge der Überlegungen hier ansehen.

  3. #3 Anonym
    28. August 2016

    Mal anders gefragt.

    Ist die Unterscheidung zwischen “träger Masse” und “schwerer Masse” vielleicht eine künstliche, die das Ganze nur unnötig kompliziert? Wenn sie immer äquivalent sind, ist das zwar egal, aber trotzdem wäre alles doch einfacher, wenn man beides einfach “Masse” nennen würde. Das spezifiziert die diesbezügliche Eigenschaft des Körpers doch hinreichend, oder nicht?

    (Bitte klärt mich mal auf. Ich hab nur Abitur, und Physik in der 11. Klasse abgelegt.)

  4. #4 KarlH
    29. August 2016

    @Anomym
    Die Unterscheidung der trägen Masse von der schweren Masse ist zunächst eine begriffliche. Die träge Masse gibt nach dem 2. Newton’schen Gesetz den Widerstand an, den ein Körper einer Beschleunigung durch eine beliebige Kraft entgegensetzt. Die Masse, die in dieser Gleichung steht, ist die träge Masse. Die Massen, die im Newton’schen Gravitationsgesetz stehen, sind die schweren Massen der beteiligten Körper. Das Erstaunliche ist, dass beide Massen gleich gesetzt werden können. In beiden Gesetzen kann die gleiche (träge oder schwere) Masse verwendet werden, was ja auch in der Physik getan wird. Es wäre durchaus denkbar, dass es in der schweren Masse Materialunterschiede gibt. Zwei Körper mit der gleichen trägen Masse (z.B. 1kg) aus verschiedenen Materialien (z.B. Eisen und Blei) könnten verschiedene schwere Massen haben. Dies ist aber nicht der Fall, wie empirisch mit sehr hoher Genauigkeit gezeigt wurde. Die Konsequenz dieser Äquivalenz der trägen und schweren Masse ist, dass der zeitlich-räumliche Bewegungsablauf eines Körpers in beliebigen Schwerefeldern überhaupt unabhängig von Masse ist. Dies ist der Ausgangspunkt in Einsteins Allgemeiner Relativitätstheorie in der die gerade und gleichförmige Bewegung des ersten Newton’schen Gesetzes bei fehlender Kraft durch die (i.A. gekrümmte) Bahn bei Anwesenheit eines Gravitationsfeldes ersetzt wird. Damit ist die Gravitation bei Einstein keine Kraft mehr sondern eine Wirkung auf die Geometrie von Raum und Zeit.

  5. #5 MartinB
    29. August 2016

    @Anonym
    KarlH hat es sehr gut erklärt – nur eine kleine Ergänzung: Man *muss* die ART nicht als Theorie der Raumzeitkrümmung formulieren, man kann sie auch als eine Theorie verstehen, in der es ein Gravitationsfeld ähnlich zu einem elektromagnetischen Feld gibt, das genau so wirkt, dass für uns der Eindruck einer gekrümmten Raumzeit entsteht.

  6. #6 Anonym
    29. August 2016

    Vielen Dank @ KarlH und MartinB
    Die Unterschiede in den Definitionen sind mir jetzt klar, zumindest klarer als vorher.

    Aber täusche ich mich, oder hat noch keiner erklärt, warum der Begriff “Masse” nicht genügt? Ich kann ja notfalls statt “träger Masse” auch sagen “Masse, auf die eine Kraft einwirkt”, damit erkennbar ist, dass die “schwere” Masse jetzt durch eine einwirkende Kraft in Bewegung versetzt wird. Wobei dann eine Gegenkraft auftritt, die vorher nicht da war, und die ich von vornherein aus der schweren Masse, wenn ich sie kenne, berechnen kann, wenn ich auch die einwirkende Kraft kenne. Die Rechnung ist sogar ziemlich einfach, weil Kraft = Gegenkraft.

    ich finde das alles viel logischer, wenn es nur “Masse” heißt. Ob es schwere Masse oder träge Masse ist, kann ich doch aus dem Zusammenhang ableiten, von dem die Rede ist, oder nicht?

    Ich finde das genauso bescheuert, wie sie mir auf der Berufsschule die Buchhaltung beigebracht haben. Die fangen mit Buchungssätzen an. Sachverhalt: Karl zahlt 10 € an Firma. Im Buchungssatz der Firma steht dann “Kasse an Karl 10 €”. Das ist nicht logisch.

    Logisch ist es erst von hinten her betrachtet, weil alles in der Buchführung am Ende in die Bilanz muss. Und da muss es auf der richtigen Seite (Soll oder Haben) stehen. Und damit es da steht, ist der Buchungssatz genau verkehrt herum.

    Wenn sie es mir gleich richtig beigebracht hätten, hätte ich vielleicht keine zwei Jahre gebraucht, um das zu kapieren und deshalb nicht fast meine Lehre schmeißen müssen.

  7. #7 Anonym
    29. August 2016

    Vielen Dank @ KarlH und MartinB
    Die Unterschiede in den Definitionen sind mir jetzt klar, zumindest klarer als vorher.

    Aber täusche ich mich, oder hat noch keiner erklärt, warum der Begriff “Masse” nicht genügt? Ich kann ja notfalls statt “träger Masse” auch sagen “Masse, auf die eine Kraft einwirkt”, damit erkennbar ist, dass die “schwere” Masse jetzt durch eine einwirkende Kraft in Bewegung versetzt wird. Wobei dann eine Gegenkraft auftritt, die vorher nicht da war, und die ich von vornherein aus der schweren Masse, wenn ich sie kenne, berechnen kann, wenn ich auch die einwirkende Kraft kenne. Die Rechnung ist sogar ziemlich einfach, weil Kraft = Gegenkraft.

    ich finde das alles viel logischer, wenn es nur “Masse” heißt. Ob es schwere Masse oder träge Masse ist, kann ich doch aus dem Zusammenhang ableiten, von dem die Rede ist, oder nicht?

    Ich finde das genauso bescheuert, wie sie mir auf der Berufsschule die Buchhaltung beigebracht haben. Die fangen mit Buchungssätzen an. Sachverhalt: Karl zahlt 10 € an Firma. Im Buchungssatz der Firma steht dann “Kasse an Karl 10 €”. Das ist nicht logisch.

    Logisch ist es erst von hinten her betrachtet, weil alles in der Buchführung am Ende in die Bilanz muss. Und da muss es auf der richtigen Seite (Soll oder Haben) stehen. Und damit es da steht, ist der Buchungssatz genau verkehrt herum.

    Wenn sie es mir gleich richtig beigebracht hätten, hätte ich vielleicht keine zwei Jahre gebraucht, um das zu kapieren und deshalb nicht fast meine Lehre schmeißen müssen.

  8. #8 MartinB
    29. August 2016

    @Anonym
    Meist sagt man ja auch schlicht “Masse”, den Unterschied träge/schwere Masse macht man ja eigentlich nur dann, wenn man auf die unterschiedlichen Konzepte (Masse als Größe, die Schwerefelder erzeugt und auf sie reagiert bzw. Masse als Größe, die sagt, wieviel Beschleunigung ich pro Kraft bekomme) hinweisen will.
    Und gerade der Massebegriff ist eh etwas überfrachtet:
    https://scienceblogs.de/hier-wohnen-drachen/2012/07/25/die-vielen-gesichter-der-masse/

  9. #9 Aveneer
    29. August 2016

    “…dass für uns der Eindruck einer gekrümmten Raumzeit entsteht.”
    Daumen rauf für einen solchen Kommentar.

  10. #10 Aveneer
    29. August 2016

    Träge und schwere Masse – War schon öfters ein Thema.
    Das „Machsches Prinzip“ ist für mich “im Prinzip” nicht ganz vom Tisch.
    Im Besonderen, wenn man eine Art Feld als Ersatz für die Raumzeit nicht ausschließen kann.

  11. #11 MartinB
    29. August 2016

    @Aveneer
    Das Feld “ersetzt” nicht die Raumzeit – in der Feldformulierung ist die Raumzeit der Minkowski-raum der SRT; das Feld sorgt nur für eine entsprechende verzerrung aller Maßstäbe, so dass wir einen gekrümmte Raum wahrnehmen.

  12. #12 Aveneer
    29. August 2016

    Du meinst „Dass wir eine gekrümmte Raumzeit wahrnehmen“? Oder hast du „Zeit“ mit Absicht herausgelassen?
    Könnte nicht jegliche Geometrie der Raumzeit (z.B. Minkowski-raum) nicht allein durch die intrinsische Eigenschaft eines Feldes herleitbar/begründbar sein?

    „Hilbertraum“ + Feld = „Minkowski-Raumzeit“

  13. #13 MartinB
    29. August 2016

    @Aveneer
    “Du meinst „Dass wir eine gekrümmte Raumzeit wahrnehmen“?”
    Ja, meinte ich.
    “Könnte nicht jegliche Geometrie der Raumzeit (z.B. Minkowski-raum) nicht allein durch die intrinsische Eigenschaft eines Feldes herleitbar/begründbar sein? ”
    Wie soll das gehen? “Feld” ist die Bezeichnung für eine Größe, die jedem Punkt des Raumes oder der Raumzeit definiert ist.
    Es mag natürlich irgendeine andere physikalische Größe geben, die letztlich die Eigenschaften der Raumzeit begründet – da gibt es ja Ideen wie Loop Quantum Gravity etc., aber bisher ist meines Wissens keine dieser Ideen vollkommen ausgereift.

  14. #14 Solkar
    29. August 2016

    @MartinB wg. #11

    Ist “Feldformulierung” als formaler Gegensatz zur Formulierung auf dem Tangentialbündel gemeint?

  15. #15 JoJo
    29. August 2016

    Anstatt “Einstein hat die Newton’schen Gesetze vertauscht” könnte man doch auch sagen, dass Einstein alles von einem anderen Bezugssystem aus betrachtet? (Ergibt natürlich kein so tollen Blog-Titel…)

    Im ersten Bild mit dem still hängenden Apfel betrachtet man alles ja aus einem beschleunigten Bezugssystem — etwa wenn man unbewegt neben dem Apfelbaum steht. Hier ist schon mal überraschend, dass in diesem beschleunigten Bezugssystem die Newton’schen Gesetze gelten, die ja nur für Inertialsysteme formuliert sind. In nicht-initerialen Systemen treten i.d.R. zusätzliche Scheinkräfte auf.

    Wird Apfel + Baum hingegen aus einem Inertialsystem betrachtet, ist die Situation einerseits einfacher, weil keine Scheinkräfte mehr auftreten, andererseits für Erdenmenschen eher ungewohnt, da man seinen Alltag nun mal in einem beschleunigten Bezugssystem verbringt.

    Oder würde man im Newton-Bild besser auch von einer Scheinkraft reden?

  16. #16 Solkar
    29. August 2016

    @Aveneer wg #12 “Hilbertraum“ + Feld = „Minkowski-Raumzeit“

    Die durch diag(-1, 1, ,1, 1) vermittelte Bilinearform auf dem |R^4 ist nicht positiv definit, deshalb ist der Minkowski-Raum kein Hilbertraum.

    Ein Hilbertraum erfordert qua def ein Skalarprodukt, und Skalarprodukte müssen qua def positiv definit sein.

  17. #17 MartinB
    29. August 2016

    @Solkar
    ganz ehrlich? Weiß ich nicht.
    Mit Feldformulierung ist gemeint, dass man die ART ja auch über Felder an jedem Punkt einer statischen Minkowski-Raumzeit formulieren kann (siehe das Buch von Weinberg oder Feynman) – aber im faserbündelformalismus stecke ich nicht mehr tief genug drin.

    @JoJo
    Bin mir nicht sicher, ob das ne gute Betrachtungsweise ist. Wenn du es so siehst, dann ist die Gravitation aber ja in gewisser Weise genau eine Scheinkraft. Man könnte vielleicht sagen, dass Einstein umdefiniert hat, welche Systeme Inertialsysteme sind. Ich weiß aber nicht, ob das die Erklärung wirklich einfacher macht – die habe ich neulich extra so ausgedacht, um in einem Vortrag in 5 Minuten zumindest eine Idee geben zu können, wie Einstein auf die gekrümmte Raumzeit kam.

  18. #18 Solkar
    29. August 2016

    @MartinB wg. #17: “Mit Feldformulierung ist gemeint, dass man die ART ja auch über Felder an jedem Punkt einer statischen Minkowski-Raumzeit formulieren kann
    Danke, dann weiß ich schon, was gemeint ist.

    (siehe das Buch von Weinberg oder Feynman)
    Die hab ich zwar leider beide nicht, aber MTW umreißen das ganz ordentlich in §7.1.

  19. #19 MartinB
    29. August 2016

    Der MTW erklärt das auch nochmal weiter hinten im Zusammenhang mit der Spin-2-“Herleitung”, irgendwo Kap. 17 oder 18 glaube ich.

  20. #20 Solkar
    29. August 2016

    @MartinB wg. #19
    Bei MTW, Box 7.1 findet man
    H^{μανβ}_{,αβ} = 16πGT^{μν} (4′).
    Das H^{μανβ} wäre dann mE das Feld in der Feldformulierung wobei dieses H gem. ebd. Gl (4) nur an ein Spin-2-Feld h^{μν} und den flachen η^{μν} koppelt.

    Mit ein wenig Eichung kriegt man dann daraus sogar eine simple WGl (ebd Gl (7)).

    Das ist ja soweit alles recht kompakt, aber ich frage mich, ob das nicht letztlich nur eine Näherung für schwache Felder ergibt, denn das sieht doch auf den ersten Blick sehr störungstheoretisch und linearisiert aus.

  21. #21 MartinB
    29. August 2016

    @Solkar
    Schau mal in Box 17.2, Punkt 5 und Box 18.1.
    Ja, man startet mit einer linearisierten Theorie, dann führt man Ordnung für Ordnung weitere Terme ein, um das ganze Selbstkonsistent zu machen (wegen der Selbst-Wewi der Gravitation) und landet am Ende zwangsläufig bei der Einstein-Gleichung.
    Ist z.B. in den Feynman Lectures on Gravitation so hergeleitet. (Die sind eh sehr zu empfehlen…)

  22. #22 Anonym
    29. August 2016

    @ MartinB, vielen Dank auch für den sehr interessanten Artikel, den du in #8 verlinkt hast. https://scienceblogs.de/hier-wohnen-drachen/2012/07/25/die-vielen-gesichter-der-masse/

    Der ist zwar recht lang (macht mir persönlich nichts aus), aber auch richtig gut. Ich hab ihn heute nachmittag schon mal gelesen und werde das gleich nochmal tun, damit das auch endlcih mal bei mir hängen bleibt.

  23. #23 Anonym
    29. August 2016

    Habe ich das jetzt endlich richtig kapiert?

    Wenn ich einen Körper nahe an die Lichtgeschwindigkeit beschleunigen will, dann gelingt mir das nicht, weil die “träge” (“relativistische”) Masse bei solchen Geschwindigkeiten gegen unendlich strebt.
    Wenn ich aber ein Objekt mit großer “schwerer Masse” hätte, beispielsweise einen Stern, der sich schon fast mit Lichtgeschwindigkeit bewegt, dann verändert sich durch die Bewegung dessen schwere Masse (also seine Gravitationskraft) nach außen hin überhaupt nicht. Nur weiter beschleunigen könnte man ihn nicht, weil das jenseits aller darstellbaren Kraft wäre.
    Oder einfacher: Bei “irdischen” Geschwindigkeiten sind träge und schwere Masse gleich. Nahe der Lichtgeschwindigkeit nimmt aber die träge Masse zu, während die schwere Masse gleich bleibt.
    Ist das so? (Bitte sagt mir, dass das so ist.)

  24. #24 Aveneer
    30. August 2016

    @ Anonym
    Ich denke du verwechselst Ruhemasse mit schwere Masse?
    Ruhemasse: Ist die Masse eines Teilchens in seinem Ruhesystem. Ruhemasse ist eine Besonderheit von Teilchen mit v<c. Träge/Schwere Masse hingegen kann Ruhemasse enthalten, muss es aber nicht.

    Im Ruhesystem: Ruhemasse = schwere Masse = Träge Masse

    Aber auch „reine“ Energie wie z.B. Photonen besitzen Träge Masse. "Kinetische Energie", wenn man so möchte auch. Druck, gespante Feder….Die Träge/Schwere Masse entspricht der „Gesamtenregie“. Diese wächst z.B. mit der Geschwindigkeit. (Sie ist "relativ" da z.B abhängig ist vom Bewegungszustand der Beobachter)

    Gesamtenregie/c^2= m , wobei m in der „Kinetik“ Träge Masse ist und bei „Newton bzw. RT“ schwere Masse.

  25. #25 Aveneer
    30. August 2016

    Hilbertraum war falsch (@Solkar) – ich meinte einfach einen nicht physikalischen Raum. Ein „N-Dimensionale NICHTS“. Ein Raum der keine physikalischen Vorgaben gibt. In der SRT wird zwar kein wirkendes Feld benötigt (siehe „Lorentz“), aber ohne dieses, könnte es sowas wie die Minkowski-Raumzeit ggf. nicht geben. Einfach gesagt, es begründet die Existenz von Ruhe-/Inertialsystem. Ein „Hintergrundfeld – eine Bühne“ – relevant für die QM – aber ohne eigene (Aus)wirkung in der SRT. Ich halte zumindest nichts davon, das Raumzeit-Modell in der SRT aufrecht zu erhalten, wenn es in der ART ersetzt werden könnte. (Raum)zeit ist das, was man von der Uhr abliest. Das „Hintergrundfeld“ muss ausschließlich die Existenz von Uhren begründen. Die Minkowski-Raumzeit folgt dann aus der Konstanz der LG.

  26. #26 MartinB
    30. August 2016

    @Anonym
    “(Bitte sagt mir, dass das so ist.)”
    Sorry, aber es ist nicht ganz so.
    Je schneller einObjekt wird, desto schwerer ist es zu beschleunigen, und wenn du das Konzept “relativistische Masse” verwenden willst, dann kannst du das formulieren als “Die trägheit/träge Masse nimmt immer weiter zu”.

    Um zu sehen, welches Schwerefeld sich ergibt, musst du bei Objekten nahe der Lichtgeschwindigkeit aber die Allgemeine Relativitätstheorie nehmen, nicht die Newtonsche Theorie. (Nach der hättest du recht, aber die gilt halt nicht für Objektenahe der Lichtgeschwindigkeit)
    Und die sagt dir, dass du nicht einfach – wie bei Newton – annehmen darfst, dass das Schwerefeld proportional zur Masse ist, wenn sich ein Objekt schnell bewegt, weil eben noch weitere Effekte hinzukommen. (Ähnlich wie bei bewegten elektrischen Ladungen Magnetfelder entstehen.)

    Das Gravitationsfeld ändert sich also schon durch die schnelle Bewegung, aber nicht so, wie du es nach dem Newtongesetz erwarten würdest.

  27. #27 Solkar
    30. August 2016

    @Aveneer wg #25 “Ich halte zumindest nichts davon, das Raumzeit-Modell in der SRT aufrecht zu erhalten, wenn es in der ART ersetzt werden könnte.

    Na ja – was heisst aufrechterhalten?
    Die ART “lebt”¹ zwar formal auf dem Tangentialbündel von Mannigfaltigkeiten.
    Wenn das Modelluniversum aber in allen Tangentialräumen an allen Punkten flach ist, also
    g_{μν}(t, r) = const = η_{μν} =² diag(-1, 1, 1, 1),
    dann kann man die Integration längs Weltlinien nicht mehr von der auf dem Minkowski-Raum unterscheiden, ergo erhält man auch die gleichen Werte, und damit ist in Abwesenheit von relevanter Gravitation der Minkowski-Raum als Modell gut genug.

    ¹ scheußlicher Slang
    ² cartesisch

  28. #28 Solkar
    30. August 2016

    @MartinB wg #29 Feynman Lectures on Gravitation […]. (Die sind eh sehr zu empfehlen…)

    Ja, den sollte ich mir wohl mal zulegen.
    Ich hab nur gegen Feynmans andere Lectures (also “die Lectures”, die man gemeinhin namentlich so zusammenfasst) eine tief verwurzelte Skepsis der Art cui bobo.
    Die wurden zwar immer empfohlen, ich wußte immer nicht warum ich die noch hätte anfassen sollen, nachdem ich schon meist bei Nolting und spätestens bei LL fündig geworden war.

  29. #29 Aveneer
    30. August 2016

    Gut genug – für Pragmatiker. Ich bin sehr optimistisch, dass das Modell „Raumzeit“ vollständig durch eine Feldtheorie ersetzt wird und zudem eines Tages experimentell (z.B. Beschleuniger) auch widerlegen werden kann. Ein mögliches Indiz dafür: Das Fehlen von echten physikalischen Grenzen der Raumzeit. Ein „Gummituch“ mit dem man im Grunde wirklich alles machen kann, was „der Term (die Mathematik)“ hergibt. Für ein (ich sag mal) „Lorentz-Einstein-Feld“ würden sich Grenzen ergeben (max. Krümmung? = keine Singularität?) und es lassen sich Größen berechnen (Elastizität, Widerstand….).
    Die RT ist das Modell mit den wenigsten annahmen – ich denke mindestens eine zu wenig. Aber: quod esset demonstrandum

  30. #30 Solkar
    30. August 2016

    Nein, “gut genug” für alles, was Physik überhaupt leisten können soll.

  31. #31 MartinB
    30. August 2016

    @SOlkar
    Die “normalen” Lectures sind einfach – anders. Als alleiniges Physikbuch vermutlich untauglich (außer Band II, der ist ja eher konventionell), aber voller raffinierter Ideen und Argumente, dieman sonst nirgends bekommt, und immer interessant, weil zum Mitdenken gemacht.

    Brauchst du dir aber nicht zuzulegen, gibt es komplett frei lesbar (wenn auch nicht zum download) im Netz.

  32. #32 Solkar
    30. August 2016

    @MartinB wg. #31

    Ja, beim Caltec
    https://www.feynmanlectures.caltech.edu/

    Nur ist mir Feynmans Stil insgesamt zu invasiv; ich weiß nicht, ob es Feynman klar war, dass es Gedankenwelten abseits der seinen gab.

    Ich zumindest möchte nicht Richard Feynman in meinem Kopf haben.

  33. #33 a.n
    30. August 2016

    @Aveneer (29)

    Bitte? Was ist denn eine Feldtheorie ohne “Raumzeit”?

  34. #34 Solkar
    30. August 2016

    Bitte? Was ist denn eine Feldtheorie ohne “Raumzeit”?

    42.¹

    ¹ In pseudo-natürlichen Einheiten. Das entspricht 42 femtoschwurbel im SI-System.

  35. #35 Aveneer
    30. August 2016

    @Solkar
    Zitat:
    #13 MartinB
    Es mag natürlich irgendeine andere physikalische Größe geben, die letztlich die Eigenschaften der Raumzeit begründet
    Zitat Ende.
    Diese Größe gibt es denke ich. Z.B.
    In einem Universum angefüllt mit Photonen (kein „Ruhesystem“) würde man keine Zeit definieren können – aber trotzdem Bewegung. Alles was du nun benötigst: Finde eine Größe für diese Bewegung.

  36. #36 Anonym
    30. August 2016

    @ Aveneer (#24) und MartinB (#26)

    Vielen Dank für eure Mühe.

    Aber ich kapier’s einfach trotzdem nicht so richtig. Für mich ist die Quintessenenz aus dieser Diskussion jetzt einfach mal, dass man auch einfach “Masse” (statt “schwerer” und “träger” Masse) sagen könnte, weil diese beiden “hypothetischen” Massen in der Praxis identisch mit der *Masse* eines Körpers sind. Und zwar sowohl bei Einstein als auch bei Newton. Warum das so ist, kann uns im Prinzip egal sein, das wäre eine philosophische Frage; das ist offenbar einfach so.

    Verzeiht mir bitte; aber ich steige verständnismäßig immer an der Stelle aus, an der mathematische Formeln ins Spiel kommen. Mit denen kann ich nämlich nicht umgehen. Selbst das Umstellen von Gleichungen wie R = U / I (nach I z.B.) fällt mir schwer. Ich bin sowas wie ein mathematischer Legastheniker.

    Und das ist natürlich schlecht, wenn man sich gleichzeitig für Astrophysik, Quantenphysik und relativistische Effekte interessiert.

    Seufz. Kann man nicht ändern. Bin wohl genetisch so veranlagt.

  37. #37 MartinB
    30. August 2016

    @Aveneer
    Wie soll man Bewegung (änderung des ortes mit der Zeit) ohne Zeit definieren können?

    @Anonym
    ” Für mich ist die Quintessenenz aus dieser Diskussion jetzt einfach mal, dass man auch einfach “Masse” (statt “schwerer” und “träger” Masse) sagen könnte, weil diese beiden “hypothetischen” Massen in der Praxis identisch mit der *Masse* eines Körpers sind.”
    Richtig. Theoretisch muss das nicht so sein, aber in unserem Universum ist es so.

    ” Und zwar sowohl bei Einstein als auch bei Newton.”
    Ebenfalls richtig.

    “Warum das so ist, kann uns im Prinzip egal sein, das wäre eine philosophische Frage; das ist offenbar einfach so. ”
    Jein. Bei Newton ist das einfach so, bei Einstein ist es eine notwendige Folge von dem, was ich hier im Artikel schreibe – sonst ürden unterschiedliche Objekte unterschiedlich schnell im freien fall fallen, dann könnte ich aber feststellen, ob ich im freien Fall bin oder nicht – wenn zum Beispiel Äpfel schneller fallen als Menschen, dann kann ich einen Apfel loslassen – wenn der sich von mir entfernt, weiß ich, dass ich in einem Schwerefeld falle, wenn nicht, weiß ich, dass ich mich in keinem Schwerefeld befinde.

    Die Äquivalenz von träger und schwerer Masse ist die notwendige Bedingung dafür, dass man eine Bewegung im Schwerefeld über die gekrümmte Raumzeit beschreiben kann.

    “aber ich steige verständnismäßig immer an der Stelle aus, an der mathematische Formeln ins Spiel kommen”
    Falls du sie nicht schon kennst, versuch’s doch mal mit der kleinen Artikelserie “Die Gleichungen der Physik”; links findest du hier:
    https://scienceblogs.de/hier-wohnen-drachen/artikelserien/
    Da habe ich versucht, für genau so Leute wie dich ein wenig nahezubringen, wie man mit fiesen Gleichungen umgeht. Gleichungen irgendwie gruselig zu finden und sofort beim Anblick einer Gleichung abzuschalten, ist ja keine Schande und auch nicht gerade selten.

    Und gern auch dort Fragen in die Kommentare schreiben, wenn’s irgendwo klemmt.

  38. #38 Anonym
    30. August 2016

    @ MartinB
    Danke für den Link, habe ich mir gerade gebookmarkt und vor allem den Abschnitt über Formeln werde ich mir in den nächsten Tagen verabreichen.

    Eine Frage noch. Du schreibst in #26:

    “Um zu sehen, welches Schwerefeld sich ergibt, musst du bei Objekten nahe der Lichtgeschwindigkeit aber die Allgemeine Relativitätstheorie nehmen, nicht die Newtonsche Theorie. (Nach der hättest du recht, aber die gilt halt nicht für Objektenahe der Lichtgeschwindigkeit)”

    Ok, müsste ich wohl.

    Aber bevor ich das ausrechnen kann, muss ich wohl deine Artikel über die Formeln und dann nochmal Einsteins Büchlein über die Allgemeine und die Spezielle Relativitätstheorie durcharbeiten. (Ich hab’ das im Schrank. Und er beschreibt das mit seinen Fahrstuhl- und Eisenbahn-Beispielen auch sehr anschaulich. Aber es sind halt auch Formeln drin. Einige.)

    Du weißt das doch sicher aus dem Stegreif. Wie verhält sich denn nun das Schwerefeld für den außenstehenden Beobachter, wenn ein Stern mit einer Geschwindigkeit ganz nahe an der Lichtgeschwindigkeit an ihm vorbeisaust? Es kann ja nicht gegen unendlich gehen, wie die träge Masse. Denn dann würden ja ganze Milchstraßen, die “am Weg liegen”, in den Stern gesaugt und ein riesiges rasendes Schwarzes Loch entstünde. Oder verrenne ich mich da in abstruse Gedanken?

  39. #39 Aveneer
    30. August 2016

    @MartinB
    Zitat: Wie soll man Bewegung (änderung des ortes mit der Zeit) ohne Zeit definieren können?

    Ich frage mich, ob das Elektron Bewegung definieren muss? Ob die Natur „rechnet“? Sie mag würfeln aber rechnen? Zeit benötigt man um sagen zu können, wann, was, wo ist. In der Natur gibt es nicht vielmehr als das „Jetzt“. Ein Zustand der mathematisch eigentlich nicht erreicht werden kann.

    Mittlerweile glaube ich, dass das „Wie“ so schwierig zu verstehen ist wie der Schritt von „Raum und Zeit“ zu „Raumzeit“. Bzw. Newton zur RT… Da man erst einmal verstehen muss, dass Bewegung „Ortsänderung“ die Größe Zeit nicht benötigt.
    Einfaches Beispiel: A) Snooker. Die Kugeln ändern ihre Ruhelage zueinander nicht, nur weil die Zeit vergeht – Umgekehrt werden sie eben nicht langsamer weil die Zeit langsamer vergeht. Umgekehrt ist richtig: Die Zeit vergeht langsamer, weil die Kugeln sich langsamer Bewegen (Ursache: Feld? „Induktion“ was auch immer…) oder B) (Ist ganz einfach, wenn man darüber Nachdenkt.) In z.B. einem photonischen Universum findet jedes Ereignis „absolut“ gleichzeitig statt. Da der zurückgelegte Weg ds immer ds‘ entspricht. Z.B. ein Photon fliegt von A nach B (=ds) und löst dort ein Ereignis X aus. Alle anderen Photonen haben bis dahin denselben Weg zurückgelegt (ds=ds‘). = jedes Ereignis X ist mit allen anderen Ereignissen absolut Gleichzeitig, wenn der Abstand identisch ist.
    Eine Größe Zeit ist hier nicht notwendig, da sich alles und immer im JETZT befindet (da ds=ds’). Ich nenne es mal Bewegung im ewigen JETZT. Vergangenheit und Zukunft (dt) gibt es dort nicht.
    Das gilt auch noch, wenn sich z.B. ein (paar) Photon(en) durch ein Medium wie Glas bewegt(en) oder (übertragen) ein Elektron durch ein Higgs-Feld…
    Wenn du mich fragst wie man
    Zitat „Bewegung (änderung des ortes mit der Zeit) ohne Zeit definieren können?“

    ds=ds‘ (mit der Annahme, dass Teilchen mit Ruhemasse den der Photonen im Glas(Feld) entsprechen)
    Du nimmst eine Lichtuhr und hast “ds”. Alle Ereignisse in einem Abstand ds’ sind gleichzeitig. Und wenn ein Teilchen sich durch ein Medium wie Glas bewegt oder durch ein (Higgs)Feld, dann ist der räumliche Abstand sozusagen kleiner. Die “Viererstrecke/Vierervektor” ist aber Konstannt ds=ds’…

  40. #40 MartinB
    31. August 2016

    @Anonym
    “Wie verhält sich denn nun das Schwerefeld für den außenstehenden Beobachter, wenn ein Stern mit einer Geschwindigkeit ganz nahe an der Lichtgeschwindigkeit an ihm vorbeisaust? ”
    Ich gebe ehrlich zu, dass ich das aus den Hut nicht so sagen kann. Das Schwerefeld geht nicht gegen unendlich, aber wie es genau aussieht, dafür habe ich kein Bild im Kopf, kann mich auch nicht entsinnen, das mal irgendwo vorgerechnet gesehen zu haben.

    @Aveneer
    “Ich frage mich, ob das Elektron Bewegung definieren muss? ”
    Ja, sicher, aber das ist egal. Wir wollend ie Natur beschreiben,w ir müssen das mathematsich tun, und wir beobachten Bewegung. Also muss unsere fundamentale Naturbeschreibung das Konzept irgendwie hergeben (und sei es als emergente Eigenschaft).

    “Die Zeit vergeht langsamer, weil die Kugeln sich langsamer Bewegen”
    Huh? Meinst du wegen der Zeitdilatation in der SRT? Davon merken die Kugeln selbst ja nichts.

    Was du zum photonischen Universum schreibst, halte ich für absolut falsch – Photonen beschreiben wir mit Quantenfeldern, die sind an Punkten der Raumzeit definiert, und natürlich können auch zwei Photonen nacheinander einen Ort passieren (auch wenn es ohne andere Materie keine Möglichkeit gibt, das sinnvoll zu messen).

    ” Ich nenne es mal Bewegung im ewigen JETZT. ”
    Sorry, aber das ist schlicht unsinnig – wenn es nur eine Gegenwart gibt, kann es keine Bewegung geben.

    Und der rest mit dem Vierervektor ergibt auch wenig Sinn.

    Sorry, aber Physik ist wirklich nicht so einfach…
    https://scienceblogs.de/hier-wohnen-drachen/2015/07/28/physik-anders-als-sie-aussieht/

  41. #41 Anonym
    31. August 2016

    @ MartinB (40)
    Danke für die ehrliche Antwort.

    Und es ist natürlich keine Schande, an so ein extremes, realitätsfernes und natürlich hypothetisches Szenario wie einen Stern, der an einem außenstehenden Beobachter mit einer Geschwindigkeit von fast Lichtgeschwindigkeit vorbeisaust, noch nie einen Gedanken verschwendet zu haben.
    Mir kam der Gedanke aber gestern. Vielleicht auch unter dem Einfluss der besonderen Zigarettensorte, die ich gestern geraucht habe. Die, die hierzulande schwer zu bekommen ist. Aber die bringt einen mitunter auf interessante Gedanken.

    Ich fürchte nur, ich kann mir das mit dem Graviationsfeld dieses komischen Sterns nicht selbst ausrechnen, selbst wenn ich alles nochmal konzentriert lese. Mathematik ist für mich einfach ein Buch mit sieben Siegeln.

    Ich versuch’s aber mal. Aber wenn jemand eine Idee hat, wie das ist, dann soll er sie bitte hier drunterschreiben.

  42. #42 Anonym
    31. August 2016

    Und dann ist da natürlich noch der zeitliche Aspekt.
    Wenn dieser Stern mit fast Lichtgeschwindigkeit an “am Weg liegenden Galaxien” vorbeirauscht, vergleiche ich das jetzt mal mit einem Auto, das mit hoher Geschwindigkeit eine Landstraße entlangfährt. Dann bildet sich unter dem Auto ein Sog, und Dinge (weggeworfene Zigarettenschachteln und Tempotaschentücher zum Beispiel) werden unter das Auto gezogen. Nur bewegt sich das Auto so schnell, dass diese Dinge natürlich hinterher nicht am Unterboden des Fahrzeugs hängen bleiben (selbst wenn dieser mit Klebstoff bestrichen wäre), sondern maximal hinterher auf der Fahrbahn liegen.
    Kann es also sein, dass der Stern schon wieder “weg” ist, bevor die wahnsinnig große Anziehungskraft überhaupt dazu führt, dass ihm Materie zugeführt wird?
    (Allerdings ist es ja nach Einstein wirklich so, dass die träge Masse bis nach unendlich wächst, wenn die Geschwindigkeit nahe genug an der Lichtgeschwindigkeit ist. Würde dann nicht der gesamte Inhalt des Universums eingezogen, egal wie schnell sich das Objekt mit unendlicher Masse = unendlicher Anziehungskraft bewegt? Würde überhaupt noch der leere Raum übrig bleiben oder würde alles auf einen Punkt kollabieren?
    Je mehr man über “schwere” und “träge” Masse nachdenkt, desto mehr ins Schwimmen kommt man. Das ist ein total krudes Konzept, und wer sich das ausgedacht hat, gehört brutal erschlagen.

  43. #43 MartinB
    31. August 2016

    @Anonym
    Doch, ich habe schon einen Gedanken dran verschwendet, ich habe es nur nie im Detail überlegt.
    Meiner Ansicht nach wird es durch die hohe Geschwindigkeit keine besonders große Anziehungskraft geben. Kurzes googeln wirft das hier aus:
    https://www.physicsforums.com/threads/gravity-exerted-by-a-fast-moving-object-versus-stationary-object.687833/
    Danach ist die Anziehungskraft um einen Faktor 2 höher als bei einer ruhenden Masse. Der link auf das angegebene paper scheint aber nicht zu funktionieren.

    Vermutlich hilft auch das hier weiter:
    https://www.physicsforums.com/threads/does-relativistic-mass-create-gravity.68454/

    Und mathematisch fidnest du hier was:
    https://www.quora.com/Relativity-physics-Does-relativistic-mass-have-gravity

    “Je mehr man über “schwere” und “träge” Masse nachdenkt, desto mehr ins Schwimmen kommt man. Das ist ein total krudes Konzept, und wer sich das ausgedacht hat, gehört brutal erschlagen.”
    Erstens möchte ich nicht mal rhetorsich solche Gewaltdrohungen hier lesen, zweitens, nein, die Konzepte als solche waren gut – die Natur ist nur ein bisschen komplizierter als wir am Anfang dachten.

  44. #44 Aveneer
    31. August 2016

    “Sorry, aber Physik ist wirklich nicht so einfach”
    Eigentlich wollte ich damit aufhören – Ich werde keinen anregen können, sich klar zu machen, dass Bewegung im „ewigen JETZT“ der Dauerzustand ist. Entweder bin ich ein Vollhorst 😉 oder man muss den Sprung selbst schaffen (wie ich).
    “wenn es nur eine Gegenwart gibt, kann es keine Bewegung geben.”
    Das steht mindestens so fest in „unseren“ Köpfen geschrieben – wie früher das „Zeit absolut sein muss“. Ich bin vielleicht naiv. Aber ich sehe das anders. Aus meiner Sicht ist es ein Trugschluss der aus der Verwendung der Zeit in der Mathematik resultiert (und Historisch, und Menschheit, und Bedeutung der Zeit und…bla,bal). Das ist so ähnlich wie das Paradoxon „Achilles und die Schildkröte“. Die Mathematik verwendet Zeit um den Ort zu bestimmen „wo es ist“ und ja, je kürzer das Zeitintervall desto kürzer die zurückgelegte Strecke. Daraus zu schließen, dass Zeit nun einen Einfluss auf den Bewegungszustand des Teilchens hat…JA-das ist zu kurz gesprungen.. du bist ja kein „Depp“. Aber was soll ich sagen? Die Mathematik kann das JETZT einfach nicht beschreiben und daraus schließt ihr, dass es die Zeit geben muss. Dabei wäre es so einfach? Eine einmal angestoßene Kugel bewegt sich auch dann exakt so weiter auch wenn du die Zeit entfernst. Bewegungsänderung? Nehme ein Feld zu Erklärung..
    Die „Existenz“ eines Protons ist nicht vom seinen Bewegungszustand abhängig. Es Existiert immer im JETZT – Niemals Gesten oder Morgen. Ob es ruht oder nicht – es existiert. Es fliegt an dir vorbei oder du versetzt dich in sein Ruhesystem (dann ruht es). Zwischen den beiden Zuständen gibt es keinen physikalischen Unterschied. Mathematik benötigt einen Beobachter die Natur eben nicht. Mathematiker benötigen die Zeit – die Natur (Physiker) eben nicht.
    Zitat: …halte ich für absolut falsch…
    Ein gewisser „TomS“ der in Foren sicher als Schwergewicht in Sachen „RT und VWI“ gilt und mich für …hält, hat zumindest das mir bestätigt. Aber vielleicht hatte er nicht alles bedacht.
    Meistens mach ich es (noch) schlimmer, wenn ich denke ich versuche es mathematisch…Aber, übersetzt könnte ich auch sagen, dass ein („flaches“) Feld, dass die „Raumzeit“ simulieren könnte, jedem Punkt im Raum eine Größe von „So“ (Vierer-Vektor) zuweist. Ob aber einem eine dimensionslose 1 weiter hilft?
    Sorry Martin du bist echt toll. Machst einen tollen Blog. Auf den ich immer gerne verweise. Aber vor ca. 10 Jahren habe ich für mich erkannt, dass es die physikalische Entität namens Zeit nicht gibt. Trotzdem verwende auch ich die Größe Zeit um nahzusehen was gestern war und morgen ist. Warum ist das nur kein Problem für mich? Aber für euch? Nach 10 Jahren Diskussion wird mich wohl nichts mehr „zurückholen“ können.

  45. #45 MartinB
    31. August 2016

    @Aveneer
    Ich habe ja mal eine text geschrieben zjm Thema “Warum die Zeit nicht vergeht”.

    “Aber vor ca. 10 Jahren habe ich für mich erkannt, dass es die physikalische Entität namens Zeit nicht gibt. Trotzdem verwende auch ich die Größe Zeit um nahzusehen was gestern war und morgen ist. Warum ist das nur kein Problem für mich? ”
    Weil du anscheinend gut damit klar kommst, dass du ein Konzept negierst, ohne das du nicht sinnvoll leben kannst und dass du auch nicht sinnvoll ersetzen kannst. Nicht jeder kann sein Gehirn so kompartmentalisieren…

  46. #46 Niels
    31. August 2016

    @MartinB @Anonym

    “Wie verhält sich denn nun das Schwerefeld für den außenstehenden Beobachter, wenn ein Stern mit einer Geschwindigkeit ganz nahe an der Lichtgeschwindigkeit an ihm vorbeisaust? ”
    Ich gebe ehrlich zu, dass ich das aus den Hut nicht so sagen kann. Das Schwerefeld geht nicht gegen unendlich, aber wie es genau aussieht, dafür habe ich kein Bild im Kopf, kann mich auch nicht entsinnen, das mal irgendwo vorgerechnet gesehen zu haben.

    In der ART gibt es keine relativistische Masse, der Energie-Impuls-Tensor ist die Quelle der Gravitation.
    Dort geht dann der Impuls ein.

    Ohne jetzt die Links von MartinB gelesen zu haben:
    Eine Rechnung ist da nur noch numerisch möglich. Schließlich ist in der allgemeinen Relativitätstheorie schon das Zweikörperproblem nicht mehr analytisch lösbar.
    Deswegen wird das in den Lehrbüchern ja immer zu einem Einkörpersystem vereinfacht, in dem man annimmt, dass eine Masse sehr viel größer als die andere ist. Dadurch lässt man die gravitative Wirkung der zweiten Masse schlicht wegfallen.

    So klappt es dann auch für den Beobachter, an dem der Stern vorbeifliegt:
    Wenn der Beobachter nicht gerade auch ein Stern mit ähnlicher Masse ist, vernachlässigt man schlicht die gravitative Wirkung dieses Beobachters.
    Weil es in der ART kein ausgezeichnetes Bezugssystem gibt, kann man jetzt einfach den Stern als ruhend annehmen und den Beobachter als bewegt.
    Mathematisch betrachtet man also die normale (Schwarzschild-)Raumzeit, die ein Stern erzeugt.
    Dann rechnet man mit einer Testmasse in dieser Raumzeit, die sich mit einer Anfangsgeschwindigkeit nahe der Lichtgeschwindigkeit bewegt.
    Die Bahnen für solche Testmassen sind analytisch berechenbar und bekannt.

  47. #47 MartinB
    31. August 2016

    @Niels
    In dem physicsforum-Link wird gesagt (aber nicht vorgerechnet, soweit ich auf die Schnelle sehe), dass die Anziehungskraft bei v fast c um nen Faktor 2 steigt gegenüber dem Wert bei einer ruhenden Masse.

  48. #48 Niels
    31. August 2016

    @MartinB

    Na ja, die haben aber doch nur einen Spezialfall des von mir Beschrieben berechnet?

    Also den Beobachter als Testmasse angenommen und in einer Schwarzschild-Raumzeit untersucht.

    In meiner Sprechweise haben sie einfach die Schwarzschild-Raumzeit einer (natürlich ruhenden) Masse K betrachtet und anschließend die beiden Formeln für die Transversalgeschwindigkeit im jeweiligen Eigensystem zweier Testmassen A und B verglichen.
    Testmasse A hat dabei die (Transversal-)Anfangsgeschwindigkeit v beinahe c im Ruhesystem der krümmenden Masse K, Testmasse B dagegen (Transversal-) Anfangsgeschwindigkeit v gleich Null.

    Das Ungewöhnliche ist jetzt nur, dass sie eine “aktive gravitative Masse” über die Transversalgeschwindigkeit in den Testmasse-Eigensystemen definiert haben.
    Diese “aktive gravitative Masse” kommt in der Allgemeinen Relativitätstheorie doch sonst nirgends vor, oder? Ich bin mir auch nicht wirklich sicher, ob sie anschaulich wirklich hilfreich ist.
    Diese “aktive gravitative Masse” ist für eine Testmasse mit v gegen c als Transversal-Anfangsgeschwindigkeit zweimal so groß wie für eine Testmasse mit v gleich Null.

    Das gilt aber eben nur für eine Testmasse in einer Schwarzschild-Raumzeit.
    Sobald man die Gravitationswirkung der zweiten Masse nicht mehr ignorieren kann oder die erste Masse etwas anderes als eine Schwarzschild-Raumzeit erzeugt oder man die “aktive gravitative Masse” anderweitig als mit Hilfe der Transversalgeschwindigkeit im Testmassen-Beobachter-System definiert, kommen völlig andere Ergebnisse heraus.
    Dafür kann man dann im Allgemeinen aber keine analytische Lösungs-Formel mehr aufstellen.

  49. #49 MartinB
    31. August 2016

    @Niels
    Stimmt, ich hatte nicht genau genug gelesen – ist das jetzt (per Bezugssystemwechsel) aber nicht trotzdem derselbe Fall, den Anonym meinte (ein Stern zieht schnell an mir vorbei)? Steh gerade auf’m Schlauch…

  50. #50 Niels
    31. August 2016

    @MartinB
    Mir ging es jetzt mehr um das Grundlegendere, also um Anonyms ursprüngliche Frage, ohne das Sternbeispiel:

    Bei “irdischen” Geschwindigkeiten sind träge und schwere Masse gleich. Nahe der Lichtgeschwindigkeit nimmt aber die träge Masse zu, während die schwere Masse gleich bleibt.
    Ist das so? (Bitte sagt mir, dass das so ist.)

    Deine Antwort in #43 kann man so interpretieren, dass hier ganz einfach gilt:
    Die schwere Masse bleibt nicht gleich, sondern wird doppelt so groß.

    Damit hab ich ein bisschen Bauchschmerzen.
    Das klappt als Veranschaulichung eben nur, wenn man passend definierte “aktive gravitative Massen” in geeigneten Testmassen-Eigensystemen vergleicht.
    Mit allgemeiner Relativitätstheorie und koordinatenunabhängiger Raumzeitkrümmung hat das leider gar nichts zu tun.
    Ein ruhender Stern krümmt die Raumzeit eben nicht halb so stark wie ein sich mit fast Lichtgeschwindigkeit bewegender Stern gleicher Ruhemasse.

    Jedenfalls kann man aus diesem Paper nicht einfach den von dir in genannten Schluss ziehen.

    Meiner Ansicht nach wird es durch die hohe Geschwindigkeit keine besonders große Anziehungskraft geben.

    Das gilt nur für einen Spezialfall.
    Im Allgemeinen können Impulse so in den Energie-Impuls-Tensor eingehen, dass man den Einfluss der Geschwindigkeit von Massen auf die “Gravitation” erst nach einer numerischen Analyse einschätzen kann.

    Vermutlich kann es also auch Fälle geben, in denen die Geschwindigkeit durch irgendwelche komplizierten nichtlinearen Effekte den absolut überwiegenden Anteil der Gravitationswirkung verursacht.

    Na ja, ist aber wahrscheinlich alles wieder viel zu kompliziert gedacht. 😉

  51. #51 MartinB
    1. September 2016

    @Niels
    Leider ahbe ich keinen Volltextzugriff auf das paper, aber der bei physicsforum zitierte abstract sagt doch im Kern genau das, oder nicht?
    If a heavy object with rest mass M moves past you with a velocity comparable to the speed of light, you will be attracted gravitationally towards its path as though it had an increased mass. If the relativistic increase in active gravitational mass is measured by the transverse (and longitudinal) velocities which such a moving mass induces in test particles initially at rest near its path, then we find, with this definition, that Mrel=γ(1+β2)M. Therefore, in the ultrarelativistic limit, the active gravitational mass of a moving body, measured in this way, is not γM but is approximately 2γM.

    Ich verstehe das so, dass man die Beschleunigung von Testmassen betrachtet, die anfänglich in Ruhe sind, und die werden auf die bewegte Masse hin beschleunigt, und zwar um genau den Faktor 2gamma stärker als wenn die Masse sehr langsam vorbeizieht. (Wobei aus dem abstract zumindest mir nicht so ganz klar wird, wie das genau verglichen wird, denn bei einer langsam vorbeiziehenden Masse habe ich ja mehr Zeit, um beschleunigt zu werden.)

    Das heißt sicher aber auch, dass meine Aussage
    “Meiner Ansicht nach wird es durch die hohe Geschwindigkeit keine besonders große Anziehungskraft geben. ”
    falsch ist.

    Natürlich heißt das nicht, dass die Raumzeit um die bewegte Masse irgendwie durch eine Sschwarzschild_metrik ersetzt ist, in der man M durch 2gamma M ersetzt.

  52. #52 Aveneer
    1. September 2016

    @MartinB
    Hatte nicht mehr auf dem Schirm. Zumindest zeigst du da, dass man die Zeit nicht allein durch die Verwendung von Formeln verstehen kann.
    Ich kompartimentiere nicht – ich differenziere. Zwischen Ereignissen und dem „Dasein“
    Im Grund ist es so, dass ich mit einer Art Taschenlampe (von außen) alle Teilchen im Universum auf eine Leinwand projetziere. Jeder Schattenwurf/jeder Punkt auf der Leinwand stellt ein Objekt dar, das „absolut Gleichzeitig“ mit allen andern Punkten existiert“. (Manche können zerfallen – der „Grauanteil“ bleibt aber immer gleich) Alle Objekte die einen Schatten werfen sind stetig im „Jetzt“ stellen die Gegenwart da. Unabhängig einer Relativbewegung der Objekte. Das ist die Gegenwart. Existenz ist ein Synonym für Gegenwart und “Bewegungsunabhängig”. Das ist, denke ich, mathematisch nicht darstellbar? Nicht berechenbar – aber doch unsere Realität.

    Ich kann nun einen Punkt herauspicken (einen mit stets konstanter Geschwindigkeit. Unabhängig von der Geschwindigkeit der Wechselwirkungspartner – ein masseloses Teilchen eben). Die bewegen sich stets gleich(ART mal außen vor) und eigenen sich ausgezeichnet als Maßstab um Zusammenstöße „Ereignisse“ zu berechnen. Ich benötige aber eben genau dass: Ein universellen Maßstab.
    Du schreibst, das eine äußere Beobachterin ein anderes Zeitempfinden haben könnte. Ich denke auch „ihre“ Photonen bewegen sich mit c. (sonst wären wir wieder eine Art „Ausnahme“) Nein ich denke sogar, dass es keine Dimension gibt in der für masselose Teilchen nicht gilt So=So‘. Multidimensionaler Maßstab. Nur in 3 Dimensionen gilt So=So’-x1-x2-x3 sozusagen.

  53. #53 MartinB
    1. September 2016

    @Aveneer
    Das habe ich nicht mal ansatzweise verstanden, hat aber auch mit Physik wenig zu tun, soweit ich sehe.

  54. #54 Aveneer
    1. September 2016

    Beide Teile? Teil 2 – o.k kann Blödsinn sein.

    Teil 1: Ich meine das mit der Projektion ist doch nicht schwer? Dass Objekte einen Schatten werfen auch nicht. Und wenn alle Punkte ruhen, sollte man kein Problem haben, anzuerkennen das alle Objekte gleichzeitig existieren müssen? Sonst würden Punkte verschwinden oder neu auftauchen?
    Das ist ziemlich einfache Physik nur wenig Mathematik 😉

  55. #55 MartinB
    1. September 2016

    @Aveneer
    Ja, aber was hat es mit unserer Welt zu tun? Es ist nur ein unscharfes Analogie-Bild – und das ist eben keine Physik.

  56. #56 erik||e oder wie auch immer . . . ..
    1. September 2016

    @Aveneer
    “Das ist ziemlich einfache Physik nur wenig Mathematik”
    Die Crux bei der Sache:
    Parameter Zeit: Apfelbeispiel
    Dimension Zeit: Apfelbeispiel
    Geometrie von E und m: Äquivalenzprinzip: das sind 3-D(inge)
    Wie dies in eine gemeinsame Theorie zusammen bringen?
    Diese 3 Dinge sind bereits zusammen! Wir wollen ein Form (Geometrie) beschreiben und unseren Verstand in dieses EINS eingebunden sehen! Wir wollen uns verstehen, was wir bereits sind: Wissen herausnehmen – zu verstehen (transformieren) – um dann in einem neuen Zustand im EINS weiter tätig zu sein. . . . ..

    Das sind meine Bilder, welche ich in wörtliche Sprache binde. Die nächste Stufe ist, diese in eine mathematische Geometrie (Form) zu bringen . . . ..
    Wissen entnehmen und einem anderen Raum hinzu fügen: Teil I von der Gleichungen-Reihe: Energie hinzufügen, heißt, woanders zu entnehmen: E=mc2

    . . . ..
    Zu guter Letzt muss Alles einem Peer-Review standhalten . . . ..

  57. #57 Aveneer
    1. September 2016

    Natürlich ist es unscharf. So wie 10 Ebenen hinzumalen mit dem Hinweis “Jetzt”. Hier fehlt der Hinweis, dass es Ereignisse sind. Die Probenkörper selbst verschwinden nie – sind stetig Existent… Das Block Universum ist nicht klarer. Du sprichst von Ergebnissen ich von “ununterbrochener Existenz” die kann man nicht in Scheiben schneiden noch krümmen. Sie ist zeitlos – Ereignisse nicht. Warum die reine Betrachtung von Objekten (nicht Ereignissen) unphysikalisch ist musst du wissen.
    PS:Wie kann man den “Neutrino Sachverhalt” so gut und einfach darlegen und sich trotzdem Zeit für mich nehmen.
    PPS: Die Länge der Unterhaltung mit mir liegt in deiner Hand. Vom Neutrino-Text können mehr Menschen lernen.

  58. #58 MartinB
    1. September 2016

    @Aveneer
    Sorry, da ich immer noch keinen Schimmer habe, was du eigentlich meinst, kann ich nicht mehr sinnvoll antworten.

  59. #59 Aveneer
    2. September 2016

    (Auch) Meine letzte Antwort versprochen

    A) in meinem letzten „Beitrag“ bin ich auf Deinen Artikel „Zeit vergeht nicht“ eingegangen…) Falls das dich verwirrt hat?
    B) Wir „alle“ gehen davon aus, dass kein Punkt im Raum „bevorzugt“ wird. Und aus dem Ruhesystem heraus betrachtet, ist die „eigene Existenz“ (hört sich metaphysischer an als ich will!) nun mal dauerhaft/stetig (das ist mehr als Metaphysik – sondern unwiderlegt?). Klar muss man das runterbrechen bis zu den “ET” / den Quarks bzw. überlagerten Zuständen von „irgendwas“. Aber ich schließe daraus, dass auch Bewegung (vergehen von Zeit bzw. Ortsänderung) nichts daran ändert (Existenz/Gegenwart).
    „Ich“ oder die Welle „als Störung im Feld“ existiere stetig (ist immer gegenwärtig) auch während meiner Ortsänderung. Dieser Zustand ist „ereignislos“ und alleine daher mathematisch nicht beschreibbar. Aber deshalb nicht physikalisch? In diesem Zustand befinden wir uns in der SRT. Ein wirkendes Feld benötigt man nicht. Aber auch keine Raumzeit als Bühne/Wegbereiter.
    Zustandsänderungen (Beschleunigungen) entstehen (in der QM) erst durch Wechselwirkungen (Ereignisse eben) die i.A. NUR QM-Felder bedingen. Daher denke ich könnte jede Zustandsänderung durch Felder bedingt sein. Auch die Zeitkrümmung, die einen zu Fall bringen kann – ist folge von Wechselwirkungen. Kürzester Weg (oder besser Abstand) durch das Feld (wenigsten Wechselwirkungen…).
    In einer ruhigen Minute kannst du ja mal überlegen, ob wir beide nicht doch an zwei verschiedenen Orten gleichzeitig, stetig, in einem ununterbrochen, sehr langgezogenen jjjeeeetttttzzzt bzw. in der Gegenwart leben. Und du Ereignisse die darin stattfinden getrennt davon betrachen musst.
    Du solltest nur nicht diese Gleichzeitigkeit von deiner Uhr ablesen – diese (von Ereignissen) ist relativ (auch bei mir). Anstatt von relativer Gleichzeitigkeit zu sprechen – könnte man auch sagen: Nach “Minkowski” genügt es nicht, nur auf die Uhr zu schauen. Wäre es (unsauber ausgedrückt) nicht sinnvoller hierzu z.B. den Betrag des Viervektors zu verwenden? Zwei Ereigbniss sind Gleichzeitig wenn der Betrag des Vierervektors der “auslösenden Objekte” gleich ist?

    C’est la vie
    Aveneer

  60. #60 Physik-Fan
    4. September 2016

    Ich habe den Thread mal schnell durchgesehen, vielleicht habe ich was übersehen. Das mit der Äquivalenz von schwerer und träger Masse scheint noch nicht ganz klar herausgekommen zu sein. Falls doch, schon mal sorry für die Wiederholung.

    Dass alle Körper gleich fallen, hat in der ART nichts mit dieser Äquivalenz zu tun. Es gibt in der ART keine Grav.kraft. Körper “bewegen” sich gravitativ rein kräftefrei und folgen einer Geodäte in der Raumzeit. Ihre “Bewegung” ist nur durch die Geometrie der Raumzeit bestimmt und die ist nun mal f.a. Körper dieselbe. Deswegen “bewegen” sie sich gleich (“bewegen” in Gänsefüßchen weil es in der Raumzeit streng genommen keine Bewegung gibt, sondern nur die Bahn und die liegt als Ganzes vor).

    Das ist eine Folge der trägen Masse. Was ist nun mit der schweren Masse, gibt’s die überhaupt? Antwort: Klar, es gibt sie. Damit krümmen Körper die Raumzeit!

  61. #61 MartinB
    4. September 2016

    @Physik-Fan
    Nein, dem stimme ich nicht zu – denn dann wäre es ja denkbar, dass schwere und Träge Masse nicht für alle Körper 1:1 äquivalent sind.
    Träge Masse spielt ja nicht nur beim freien Fall, sondern bei jeder Form von Beschleunigung. Der Witz ist aber ja, dass ich immer ein lokales Lorentz-System finden kann, und das geht nur wegen der Äquivalenz.

  62. #62 Physik-Fan
    5. September 2016

    @MartinB
    Nein, dem stimme ich nicht zu – denn dann wäre es ja denkbar, dass schwere und Träge Masse nicht für alle Körper 1:1 äquivalent sind.

    Das wundert mich doch etwas. Ich dachte, darüber herrscht Konsens. Die geometrische Interpretation der Gravitation bedingt eine andere Interpretation der Massenarten. Die Äquivalenz verliert ihre Bedeutung, weil es die schwere Masse als Kraftquelle nicht gibt. Es muss also einen anderen Grund dafür geben, dass sich Körper im Grav.feld gleich, unabhängig von ihrer (trägen) Masse bewegen. Der Grund dafür ist, dass die Bewegung nur durch die Geometrie der Raumzeit bestimmt ist und die ist f.a. Körper die gleiche.

    Träge Masse spielt ja nicht nur beim freien Fall, sondern bei jeder Form von Beschleunigung. Der Witz ist aber ja, dass ich immer ein lokales Lorentz-System finden kann, und das geht nur wegen der Äquivalenz.

    Meinst Du mit Lorentz-System das IS (Intertialsystem)? Klar, das IS taucht in Zusammenhang mit dem (schwachen) Äquivalenzprinzip auf, welches ja für Einstein der Ausgangspunkt für die ART war. Aber durch die Idee der Raumzeitkrümmung als Grund für die Grav.effekte findet das Äquivalenzprinzip eine Umdeutung im obigen Sinn. Einen Aspekt dieser Umdeutung der Gravitation hast Du selbst sehr schön im Artikel herausgearbeitet.

  63. #63 MartinB
    5. September 2016

    @Physiker
    “Es muss also einen anderen Grund dafür geben, dass sich Körper im Grav.feld gleich, unabhängig von ihrer (trägen) Masse bewegen. Der Grund dafür ist, dass die Bewegung nur durch die Geometrie der Raumzeit bestimmt ist und die ist f.a. Körper die gleiche.”
    Ja, aber das Argument sagt dann doch genau nichts darüber, dass die träge Masse gleich der schweren ist.

    Es gilt doch nicht nur zu erklären, warum alle Körper gleich schnell fallen, sondern auch, warum bei allen Körpern das Gravitationsfeld proportional zur trägen Masse ist, denn in der Äquivalenz steckt beides drin.

  64. #64 Physik-Fan
    5. September 2016

    @MartinB
    Es gilt doch nicht nur zu erklären, warum alle Körper gleich schnell fallen, sondern auch, warum bei allen Körpern das Gravitationsfeld proportional zur trägen Masse ist, denn in der Äquivalenz steckt beides drin.

    Es ist die Frage der Trägheitsdynamik in der ART. Sie ist ja verallgemeinert ggüb. der newtonschen Mechanik. Trägheitsfrei ist es auf der Geodäte in der Raumzeit, nicht nur bei gleichförmiger Bewegung. In gekrümmter Raumzeit folgt ein Körper, auf den keine Kraft wirkt, einfach der Krümmung. Dabei ist er im freien Fall. Es ist unerheblich, wie Raumzeit gekrümmt ist und warum. Bei der Frage, warum alle Körper gleich fallen, ist die Gravitation also unrelevant. Sie bestimmt nur wie sie fallen und das gilt dann für alle Körper.

  65. #65 Niels
    6. September 2016

    @Physik-Fan @MartinB
    Ich verstehe das Problem gerade nicht ganz. Geht es darum, welche Axiome man für die ART braucht?

    Ich werfe mal der Reihe nach ein paar Definitionen in den Raum, vielleicht hilft es ja.

    Schwaches Äquivalenzprinzip:
    Alle Körper erfahren im Gravitationsfeld unabhängig von Struktur und Zusammensetzung die gleiche Fallbeschleunigung.
    Äquivalente Formulierung:
    Die (passive) schwere und die träge Masse eines Körpers sind gleich.

    Das reicht für die ART noch nicht aus. Hier braucht man als Axiom das
    einsteinsche Äquivalenzprinzip:
    1) Es gilt das schwache Äquivalenzprinzip.
    2) Es gilt lokale Lorentzinvarianz:
    Das Ergebnis jedes lokalen Nichtgravitations-Experimentes ist unabhängig von der Geschwindigkeit des frei fallenden Bezugssystems, in dem es durchgeführt wird.
    [3) Es gilt “lokale Ortsunabhängigkeit” (local position invariance):
    Das Ergebnis jedes lokalen Nichtgravitations-Experimentes ist unabhängig davon, wo (und wann) es im Universum durchgeführt wird.]

    Es gibt auch noch das sogenannte
    starke Äquivalenzprinzip.
    Dort gelten die selben drei Dinge wie für das einsteinsche Äquivalenzprinzip, es sind jetzt allerdings auch lokale Gravitationsexperimente erlaubt.
    Die ART erfüllt das starke Äquivalenzprinzip, als Axiom wird aber nur das einsteinsche Äquivalenzprinzip benötigt.

    Man kann zeigen, dass die einzigen Gravitationstheorien, die das einsteinsche Äquivalenzprinzip erfüllen, sogenannte metrische Gravitationstheorien sind.
    Metrische Gravitationstheorie:
    A) Die Raumzeit ist mit einer symmetrischen Metrik ausgestattet.
    B) Frei fallende Testkörper folgen den Geodäten dieser Metrik.
    C) In lokal frei fallenden Bezugssystemen gelten die Gesetze der SRT.

    Physik-Fan, du bist der Meinung, aus Bedingung B) für eine metrische Gravitationstheorie (bzw. für den Spezialfall ART) folgt automatisch Postulat 1) des einsteinschen Äquivalenzprinzips?
    Auch ohne Annahme von C)?

    MartinB, deiner Ansicht nach kann man nur sagen, dass B) aus 1) und 2) folgt?
    Die Umkehrung ist also nicht gültig?

    Oder habe eure Positionen jetzt falsch verstanden und zusammengefasst?

  66. #66 Niels
    6. September 2016

    Nachtrag:

    Könnten wir in der Diskussion die Masse in aktive schwere Masse, passive schwere Masse und träge Masse aufspalten?
    Die aktive und passive schwere Masse vermischt ihr nämlich ohne Not, wenn ich eure Beiträge richtig durchschaue?

    In den Definitionen von schwachem und einsteinschem Äquivalenzprinzip taucht nämlich erst einmal nur die passive schwere Masse auf, über die aktive schwere Masse wird keine Aussage getroffen.

  67. #67 MartinB
    6. September 2016

    @Niels
    Danke, sehr schön rausgearbeitet.
    Mit ist leider nicht ganz klar, was Physik-Fan genau meint, die begriffliche Trennung aktive und passive SM ist auf jeden Fall hilfreich.

    Werd ich mir gleich mal abspeichern, weil ich vielleicht mal wieder was über die ART schreiben werde…

  68. #68 Physik-Fan
    7. September 2016

    @Niels #65
    Ich versuche gerade die von Dir genannten Begriffe, auf die mir bekannte Begriffswelt abzubilden. Ganz ist mir noch nicht gelungen, was z.T. daran liegt, dass ich die Begriffe der dahinter streckenden mathematischen Analyse nicht alle kenne. In der Analyse geht es wohl auch um minimale Bedingungen für die Raumzeitstruktur. Was ist denn eine “symmetrische” Metrik? Macht eine “unsymmetrische” Metrik physikalisch überhaupt Sinn?

    Das Schwache Äquivalenzprinzip kenne ich als Fallgesetz von Galilei. Ausgangspunkt der ART war für Einstein das “Äquivalenzprinzip”, wie er es nannte und das danach zum “Schwachen” Äquivalenzprinzip wurde:

    Ein in einem homogenen Grav.feld frei fallendes System entspricht einem Inertialsystem.

    Wenn ich es richtig verstehe, ist dann das von Dir genannte Schwache Äquivalenzprinzip eine Verallgemeinerung auf das allg. Grav.feld, d.h. auch das inhomogene.

    Jetzt zu Deiner Frage. Ich sehe die Schlussfolgerung nicht so, wie Du fragst, sondern umgekehrt, genauso wie Du es darstellst. Zunächst dachte ich, dass die Geodäte mehr ein Postulat ist, allerdings ein sehr plausibles. Wie soll die Bahn eines kräftefreien Testkörpers in gekrümmter Raumzeit denn sein? Er sollte doch nur der Krümmung folgen, ohne Eigenabweichung. Daraus folgt bereits die Geodäte, soviel wie ich weiß. Das ist eine rein geometrische Betrachtung.

    Unschwer ist aber zu sehen, dass schon gemäß dem ursprünglichen Äquivalenzprinzip beim homogenen Grav.feld die Fallbahn eine Geodäte ist. Gemäß Deinen Ausführungen gilt es auch für das allg. Grav.feld, schöne Sache.

    Übrigens, so richtig klar ist mir nicht, was das “lokal” eigentl. bringt (lokale Lorentzinvarianz etc.). In einer inhomogenen Grav.feldumgebung gibt’s überhaupt keine Fallstrecke, wo das System intertial ist. Es treten immer z.B. Gezeitenkräfte auf. Klar, man kann die Fallstrecke infinitesimal machen, alle Querkräfte zu null setzen und dann integrieren. Aber dann propagiert man doch einen Fall, der physikalisch gar nicht existiert. Wie ist die richtige Argumentation?

  69. #69 MartinB
    7. September 2016

    @Physik-Fan
    Soweit ich das sehe (Niels möge mich korrigieren) bedeutet das “lokal”, dass man immer einen “flachen” Raumzeit-Schnipsel um jeden punkt der Geodäte finden kann, in dem alle Abweichungen vom Lorentz-Verhalten kleiner sind als eine vorgegebene Schranke. MaW, ich kann an jeden Punkt lokal einen Tangentialraum anlegen, die Abweichungen davon sind immer Terme zweiter oder höhere Ordnung.

  70. #70 Physik-Fan
    7. September 2016

    @Niels #66
    Zunächst, mehr formell, zur Klärung, was aktive und passive schwere Masse bedeuten: Die aktive schwere Masse ist diejenige, welche die Raumzeitkrümmung hervorruft. Die passive schwere Masse durchläuft diese Raumzeitkrümmung. Korrekt?

    Wir reden hier (gleiche Fallkurven f.a. Massen) über die träge Masse. Ich hab’ mal überlegt, wie die passive schwere Masse relevant sein könnte. Auch sie krümmt die Raumzeit, d.h. es liegt eine Überlagerung beider Krümmungen vor. Allerdings sind wir, das ist ziemlich klar, implizit von Testmassen ausgegangen, die so klein sind, dass deren eigene Raumzeitkrümmung vernachlässigt werden kann. Wenn man z.B die Sonnen eines Doppelsternsystems hernimmt, dann ist es natürlich ganz anders.

  71. #71 Niels
    7. September 2016

    @Physik-Fan

    Was ist denn eine “symmetrische” Metrik?

    Es gilt g(x,y)=g(y,x) mit x und y Tangentialvektoren.
    Bzw. für die Eintrage des metrischen Tensors gilt g_ij = g_ji.

    Macht eine “unsymmetrische” Metrik physikalisch überhaupt Sinn?

    Wenn man die Gültigkeit des einsteinschen Äquivalenzprinzips voraussetzt ergibt das physikalisch keinen Sinn. Das ist gerade der Witz.
    War für die aktuelle Diskussion aber vermutlich ein bisschen Overkill. Sorry.

    Wenn ich es richtig verstehe, ist dann das von Dir genannte Schwache Äquivalenzprinzip eine Verallgemeinerung auf das allg. Grav.feld, d.h. auch das inhomogene.

    Meines Wissens ist die von mir genannte Definition die Standarddefinition für das schwache Äquivalenzprinzip.
    Aber ich wäre auch nicht auf die Idee gekommen, die Galileischen Fallgesetze auf homogene Gravitationsfelder einzuschränken.
    Die englische Wiki definiert das auch nicht anders:
    https://en.wikipedia.org/wiki/Equivalence_principle#The_weak_equivalence_principle

    Ausgangspunkt der ART war für Einstein das “Äquivalenzprinzip”, wie er es nannte und das danach zum “Schwachen” Äquivalenzprinzip wurde

    Ich weiß nicht, ob es wirklich hilfreich ist, darüber zu sprechen, wie Einstein Begriffe verwendete und wie er auf Ideen kam.
    Tatsache ist jedenfalls, dass als Axiom der ART das schwache Äquivalenzprinzip allein nicht ausreicht.
    Man benötigt das einsteinsche Äquivalenzprinzip, das man genau deswegen auch “einsteinsch” nennt.

    Einsteinsches Äquivalenzprinzip aus Wiki-Artikel von oben:
    The outcome of any local non-gravitational experiment in a freely falling laboratory is independent of the velocity of the laboratory and its location in spacetime.

    Übrigens, so richtig klar ist mir nicht, was das “lokal” eigentl. bringt (lokale Lorentzinvarianz etc.).

    Siehe MartinB. Genau um das von ihm Beschriebene machen zu können braucht man überhaupt erst den ganzen mathematischen Ap­pa­rat mit den pseudoriemannschen Mannigfaltigkeiten usw.

    Zunächst, mehr formell, zur Klärung, was aktive und passive schwere Masse bedeuten: Die aktive schwere Masse ist diejenige, welche die Raumzeitkrümmung hervorruft. Die passive schwere Masse durchläuft diese Raumzeitkrümmung.

    Jein. Das sind wir eigentlich schon einen Schritt zu weit und gehen von der Gültigkeit der ART aus.
    Die aktive schwere Masse ist deshalb erst einmal einfach die Masse, die die Gravitationskraft ausübt.
    Die passive schwere Masse ist die Masse, auf die diese Gravitationskraft wirkt.
    Siehe wieder die englische Wiki:
    https://en.wikipedia.org/wiki/Equivalence_principle#Active.2C_passive.2C_and_inertial_masses

    Wie soll die Bahn eines kräftefreien Testkörpers in gekrümmter Raumzeit denn sein? Er sollte doch nur der Krümmung folgen, ohne Eigenabweichung. Daraus folgt bereits die Geodäte, soviel wie ich weiß.

    Das ist eigentlich keine Folgerung sondern schlicht die Definition von Geodäte.

    Zunächst dachte ich, dass die Geodäte mehr ein Postulat ist, allerdings ein sehr plausibles.

    Ist halt die Frage, was man postulieren möchte.

    In der ART macht man das üblicherweise in “philosophischer” Weise, d.h. man setzt Äquivalenzprinzip und Relativitätsprinzip voraus und zeigt, dass daraus (mit bestimmten Zusatzannahmen) die mathematische Beschreibung der Raumzeit als Mannigfaltigkeit und schließlich die einsteinschen Feldgleichungen als mathematische Beschreibung der Gravitation folgen.
    (Ob man aus dieser mathematischen Beschreibung schließt, dass da tatsächlich etwas gekrümmt wird, oder ob man das Ganze nicht doch wieder mit Hilfe von Feldern interpretiert, ist anschließend noch einmal eine völlig andere Frage.)

    Man könnte aber natürlich auch die Mathematik postulieren (eine Raumzeit sei definiert als…), dann die Wirkung der ART postulieren und daraus dann mit dem Hamiltonsches Prinzip die einsteinschen Feldgleichungen ableiten.

    Das macht man im Prinzip so bei der Quantenmechanik.
    Da sind die Postulate ja nicht etwa “philosophische Aspekte” wie die Unschärferelation, der Welle-Teilchen-Dualismus, …
    Sondern Dinge wie
    Jedem (isolierten) physikalischen System ist ein Hilbertraum zugeordnet. Das physikalische System ist vollständig durch einen normierten Zustandsvektor aus diesem Hilbertraum beschrieben.
    […]
    Observablen werden durch hermitesche Operatoren auf diesem Hilbertraum dargestellt.
    […]

    Ich sehe die Schlussfolgerung nicht so, wie Du fragst, sondern umgekehrt, genauso wie Du es darstellst.

    Aha?
    Worüber bist du dir dann mit MartinB uneinig?

  72. #72 MartinB
    7. September 2016

    @Niels
    Sehr schön auseinandergedröselt.

    @Physik-Fan
    Es hilft in meinen Augen auch immer, wenn man sich vor Augen führt, dass man die ART nicht mit Begriffen wie “Raumzeitkrümmung” beschreiben *muss*, sondern dass es auch eine Feldinterretation auf einem (effektiv unbeobachtbarem) Minkowski-Raum gibt. Da ist dann auch klar, was aktive und passive schwere Masse im Gegensatz zu träger Masse sind. (Wobei, wen ich es richtig sehe, die Äquivalenz von aktiver und passiver schwerer Masse aus dem 3. newtonschen Axiom folgt, oder?)

  73. #73 WolfgangS
    Grafenau
    7. September 2016

    Bin leider erst heute mangels “Zeit” auf diese Diskussion gestossen.
    @ Aveneer
    hi Aveneer, Mensch (Bruder, Schwester?) im Geiste; es ist nicht nur kein Problem für Dich, auch ich hänge diesen, Deinen Überlegungen auch seit fast

    10 Jahren an und kann mir gut vorstellen, dass die Zeit als physikalische Entität nicht existiert. Nach Einstein ist die Zeit ohnehin nicht mehr da A & O im

    Universum, sondern die Geschwindigkeit (c), also Bewegung. Die Annahme der Zeit sehe ich als geniale Hilfskonstruktion, die das Leben (und

    rechnen) sehr vereinfacht und man dadurch nicht ständig auf ein kosmoligisches GPS-System zurückgreifen muss um irgemdwelche Verabredungen zu

    treffen. Auch ich habe, von dem was ich negiere, oft viel zu wenig (und es wird mit meinen 72 Lenzen rapide weniger :)). Leider komme ich mit den

    Matheprofis hier, denen ich allerdings immer gebannt und bewundernd “zuhöre”, nur ansatzweise mit, komme aus der chemischen Ecke und so habe

    ich mich schon vor Jahren gefragt, wie kann mein “Bruder (oder Schwester)” in einem schnellen Raumschiff weniger schnell graue Haare kriegen, also

    altern. Graue Haare (genauso Falten, faulende Äpfel, rostendes Eisen) sind doch chemische Prozesse, Prozesse, die nur dann ablaufen, wenn

    Teilchen sich auch treffen. In der Nähe der Lichtgeschwindigkeit gibt es aber kaum noch Lateralbewegungen (Vektoraddition) und das Ein-und

    Überholen wird auch immer schwerer, sodass es kaum noch zu Kollisionen und damit Reaktionen kommt und damit zu einer Verlangsamung, bzw.

    letzendlich Stillstand, des Alterungsprozesses (rostendes Eisen ist auch ein Alterungs(Änderungs-)prozess). Mit der Erklärung der Zeitdilatation im

    Schwerefeld tue ich mich allerdings noch etwas schwer, allerdings denke ich, dass sich Snooker-Kugeln aus Eisen mit einem Magneten unter dem

    Tisch auch nicht mehr so elegant bewegen und es dadurch wohl auch zu weniger Kollisionen kommt. –Komme gerade aus der Küche von meiner

    Zeitmaschine, habe mir einen kalten Sekt eingeschüttet, meiner Milch, die leider außerhalb der Maschine stand, ist wohl die “Zeit” davon gelaufen–
    So wie extreme Geschwindigkeiten führen auch geringe Temperaturen (auch wieder Kinetik, Bewegung) zu weniger Kollisionen und damit zur

    Verlangsamung von (Alterungs)-reaktionen. Allerdings, wenn ich mich “morgen” für 10:00 Uhr verabrede, bemühe ich natürlich auch nicht den Stand der

    Erde zur Sonne bzw. den Grad der Erdrotation, sondern folge einfach der “Bewegung” der Zeiger meiner Uhr. Nichtsdestotrotz halte ich Zeit für eine

    tolle Erfindung die das Leben sehr erleichtert.
    Ob ich damit richtig liege, sei dahingestellt, aber für mich, der Mathematik in diesem Bereich nicht soo zugänglich, ist es ein guter Anker.
    Freue mich “weiterhin” auf viele interessante Beiträge dieses tollen Forums!

  74. #74 Physik-Fan
    15. September 2016

    @MartinB
    @Niels

    Ich habe mal betreffend Feldinterpretation der ART gestöbert und etwas in “Raum-Zeit …” von Carrier reingesehen. Carrier setzt sich auf philosophischer Ebene damit auseinander und plädiert für die Feldinterpretation ggüb. der ART. Mit den recht tiefgründigen Argumenten habe mich jetzt nicht auseinandergesetzt, sondern mehr danach geschaut, was die Feldinterpretation überhaupt ist. Es gibt darin eine minkowskische Hintergrundgeometrie mit statischer Metrik, die aber nicht beobachtbar ist (komisch, bei geringem Grav.feld ist die Geometrie doch messbar mit guter Näherung minkowskisch). Die Grav.effekte kommen dadurch zustande, dass von den aktiven schweren Massen Kräfte ausgehen, die Körper verzerren und Uhren anders gehen lassen.

    Ich empfinde diesen Ansatz als künstlich (zugegeben, kann an meiner rudimentären Kenntnis liegen). Man führt erstens was nicht Beobachtbares ein und zweitens stellt sich die Frage, was denn das für Kräfte sind. Da erscheint es mir bei Einstein einfacher und eleganter zu sein, alles auf Geometrie zurückzuführen. Die Geodäte ergibt sich so praktisch von selbst und das Fallgesetz auch. Natürlich bleibt die Frage offen, wie Massen die Raumzeit krümmen, aber genauso gut sind die feldtheoretischen Kräfte rein postuliert. Sind Kräfte, die universell Körper stauchen und Uhren bremsen, überhaupt plausibel? Komisch sind sie jedenfalls. Handelt es sich dabei eigentl. um klassische Kräfte? Wenn ja, wäre das unbefriedigend, sie müssten quantentheoretisch formuliert sein. Welche Eichgruppe(n) sollen dann es sein?

    Aber lassen wir das mal dahingestellt. Der entscheidende Punkt für mich ist, ob die Feldformulierung was Neues bringt, genauer gesagt Abweichungen von der ART. Dann könnte man experimentell darüber entscheiden. Das wäre ein substantieller Fortschritt. Wenn die Feldformulierung aber empirisch äquivalent zur ART ist, was bringt es dann? Man könnte gleich bei der ART bleiben oder meinetwegen zwischen den Interpretationen hin und her wechseln, je nachdem welche besser passt. Jedenfalls gäbe es kein Argument dafür, nicht mehr von Raumzeitkrümmung zu reden. Wem die einsteinsche Begriffswelt besser erscheint, könnte sie mit voller Berechtigung weiter verwenden.

    Bringt die Feldformulierung Fortschritte bei der Quantengravitation? Immerhin ist Feldformulierung eine Kräftetheorie und da kann man es schon erwarten. Gibt’s Ansätze in dieser Richtung?

  75. #75 MartinB
    16. September 2016

    @Physik-Fan
    ich habe die Feldinterpretation ja mal ausfürhlich behandelt:
    https://scienceblogs.de/hier-wohnen-drachen/2015/03/07/ist-die-raumzeit-gekruemmt-teil-i-der-raum/
    Findest du mit den 3 Teilen unter den Artikelserien.
    (Auch mein Artikel über Quantengravitation könnte weiterhelfen).

    Ob die beiden empirisch äquivalent sind, ist scher zu sagen. Soweit ich es sehe, ist in der Feldformulierung eine echte Topologieänderung unmöglich – in er ART kann man theoretisch eine Krümmung der Raumzeit als Blase abspalten, das geht meiner Ansicht nach mit der Feldtheorie nicht. Wurmlöcher gehen auch nicht (ehrlich gesagt habe ich aber nie verstanden, wie in der RZ-Krümmungsdarstellung entschieden werden soll, zwischen welchen Punkten sich ein Wurmloch ausbildet, weil man dazu ja eine übergeordnete Metrik oder sowas bräuchte.)
    Vorteil der Felddarstellung ist, dass sie z.B. die Stringtheorie ermöglicht – da macht man Theorie im Minkowski-Raum und bekommt durch das Graviton automatisch die Raumzeitkrümmung. Von dem klitzekleinen Haken abgesehen, dass der Raum 10 Dimensionen hat, was nicht so ganz zu den Messungen passt…

  76. #76 Niels
    16. September 2016

    @Physik-Fan

    Es gibt darin eine minkowskische Hintergrundgeometrie mit statischer Metrik, die aber nicht beobachtbar ist (komisch, bei geringem Grav.feld ist die Geometrie doch messbar mit guter Näherung minkowskisch)

    Diese lokale Minkowski-Geometrie muss allerdings nicht mit der globalen Hintergrund-Geometrie identisch sein.

    Man führt erstens was nicht Beobachtbares ein und zweitens stellt sich die Frage, was denn das für Kräfte sind. Da erscheint es mir bei Einstein einfacher und eleganter zu sein, alles auf Geometrie zurückzuführen.

    Na ja, alle anderen Wechselwirkungen werden eben mit Hilfe von Kräfte beschrieben. Da ist es schon naheliegend, dass das in irgend einer Form auch für die Gravitation gelten sollte.

    Natürlich bleibt die Frage offen, wie Massen die Raumzeit krümmen, aber genauso gut sind die feldtheoretischen Kräfte rein postuliert.
    […]
    Handelt es sich dabei eigentl. um klassische Kräfte? Wenn ja, wäre das unbefriedigend, sie müssten quantentheoretisch formuliert sein. Welche Eichgruppe(n) sollen dann es sein?

    Wie Massen die Raumzeit krümmen wird doch vollständig durch die Einsteinschen Feldgleichungen beantwortet.
    Warum ist im Gegensatz dazu eine Frage, die die grundlegendsten physikalischen Theorien nie beantworten können. Dafür ist die Physik sozusagen nicht zuständig, sie beschreibt nur.

    Die Eichgruppe ist SO(1,3), natürlich klappt es bisher nur klassisch. Sonst wäre das Ganze ja auch nicht äquivalent zur ART, die ja eine klassische Theorie ist.
    Und natürlich ist das unbefriedigend, die Quantisierung ist das Ziel der theoretischen Physik mindestens der letzten 50 Jahre.

    Bringt die Feldformulierung Fortschritte bei der Quantengravitation? Immerhin ist Feldformulierung eine Kräftetheorie und da kann man es schon erwarten. Gibt’s Ansätze in dieser Richtung?

    Ich schreibe jetzt mal keinen längeren Text, sondern fasse meine Meinung einfach mal (ein bisschen unfair) zusammen:
    Nö, bringt bisher nicht besonders viel. 😉

    Carrier setzt sich auf philosophischer Ebene damit auseinander und plädiert für die Feldinterpretation ggüb. der ART.

    Bei aller Liebe für Wissenschaftsphilosophen, meiner Meinung nach hat die Geschichte der Naturwissenschaften gezeigt, dass solche Fragen letztlich empirisch beantwortet werden müssen, nicht durch Nachdenken im stillen Kämmerlein.
    Der Natur ist nämlich offenbar ziemlich egal, was wir Menschen für ontologisch schlüssig halten.
    (Ist eigentlich auch nicht besonders überraschend, unser Gehirn wurde eben für das Verständnis von Vorgängen im Energiebereich der klassischen Mechanik entwickelt und optimiert.
    Intuitives Verständnis für Prozesse auf Quantenfeldebene oder in gekrümmten Raumzeiten ist da eher ein evolutionärer Nachteil. :-))

    Das ist jetzt vielleicht ein bisschen unfair, aber ich finde, dass sich reine Philosophen bei Aussagen zur ART, Quantenfeldtheorie oder gar zu Quantengravitationstheorien wie der Stringtheorie viel zu weit aus dem Fenster lehnen.
    Meiner Ansicht ist die Grundvoraussetzung dafür nämlich, dass man mindestens die dahinter stehende Mathematik hundertprozentig verstanden hat. Und dann ist es immer noch ein weiter Weg dahin zu erfassen, was das Ganze jetzt physikalisch bedeutet.
    Das schaffen auf diesen Gebieten schon nicht besonders viele Physiker und Mathematiker, selbst nachdem sie sich damit jahrzehntelang auseinandergesetzt haben.
    Bei jemand, der an der Uni nur Philosophie studiert hat, sehe ich da eigentlich keine realistischen Chancen.

    Lange Rede, kurzer Sinn:
    Bezüglich der Entscheidung zwischen Feldinterpretation und Krümmungsinterpretation der ART ist die richtige Antwort eigentlich Feynmans berühmtes “shut up and calculate.”
    Wir wissen schon lange, dass die ART auch nur eine für bestimmte Energiebereiche gültige Näherung ist, die übergeordnete Theorie haben wir aber schlicht noch nicht gefunden.
    Welche ontologischen Schlussfolgerungen die zu findende Theorie erlauben wird ist pure Spekulation, da kann man bestenfalls Glück beim Raten haben.
    Soweit ich es sehe müsste es aber auf etwas derart Neuartiges hinauslaufen, dass da kein Philosoph schon vorher zufällig im stillen Kämmerlein drüberstolpern könnte.

    Beim momentanen mathematischen Stand, also ausgehend von der ART, finde ich, dass die Krümmungsinterpretation mit den wenigsten Zusatzannahmen auskommt.
    Wenn man sich bei solchen Dingen über die Feinheiten von Ockhams Rasiermesser streitet, sieht man aber schon, dass das Ganze auf eine Geschmacksfrage hinausläuft.

    Wenn die Feldformulierung aber empirisch äquivalent zur ART ist, was bringt es dann? Man könnte gleich bei der ART bleiben oder meinetwegen zwischen den Interpretationen hin und her wechseln, je nachdem welche besser passt. Jedenfalls gäbe es kein Argument dafür, nicht mehr von Raumzeitkrümmung zu reden. Wem die einsteinsche Begriffswelt besser erscheint, könnte sie mit voller Berechtigung weiter verwenden.

    Klar, alles richtig.
    Eine andere Sichtweise hilft aber oft beim Verständnis, “shut up and calculate” ist eben auch nicht besonders befriedigend.

    @MartinB

    ehrlich gesagt habe ich aber nie verstanden, wie in der RZ-Krümmungsdarstellung entschieden werden soll, zwischen welchen Punkten sich ein Wurmloch ausbildet, weil man dazu ja eine übergeordnete Metrik oder sowas bräuchte

    Meines Wissens ist das ein bisher ungelöstes Problem.
    Da gibt es zwei Schulen.
    Die eine sagt, derartige Topologieänderungen sind grundsätzlich unmöglich, man kann also höchsten schon beim Urknall entstandene Wurmlöcher manipulieren.
    Die andere glaubt an die Möglichkeit von Topologieänderungen und veröffentlicht dazu immer mal wieder wilde Paper, darüber wie und warum das abläuft gibt es keinen Konsens.
    Man muss auf jeden Fall zusätzliche mathematische Strukturen einführen, da gibt es aber viele Möglichkeiten, und wenn sie schon dabei sind setzen sie auch gleich nocch irgendwelche Dinge aus Stringtheorie oder Schleifenquantengravitation voraus.

    Ich könnte mir gut vorstellen, dass es mit Hilfe solcher Tricks auch irgendwie bei der Felddarstellung klappt. Da kenne ich mich aber überhaupt nicht aus.

  77. #77 MartinB
    16. September 2016

    @Niels
    Danke, du weisst mal wieder alles…
    Hast du vielleicht eine Referemz auf so ein “wildes paper” griffbereit? Würde mich ja mal interessieren, wie sowas geht (falls man da irgendwas versteht).

  78. #78 Niels
    22. September 2016

    Sorry, im Moment nicht.
    Ich hab zwar einen überquellenden Ordner mit Papern, die ich mir bei Gelegenheit mal anschauen will (seltsamerweise wird der größer und immer größer…), aber da finde ich nichts zu diesem Thema.
    Ist schon ne Weile her, dass ich mich damit beschäftigt habe und ich erinnere mich nicht mehr richtig, wo ich auf diese Arbeiten gestoßen bin oder wie sie hießen.
    Ich glaube aber, es lief irgendwie über einen Thread bei den physicsforums?

  79. #79 MartinB
    23. September 2016

    @Niels
    danke, hätte ja sein können, dass du das griffbereit oder im Kopf hast (Irgendwie denke ich ja immer, dass du *alle* paper zu Interpretationen der QM und zur ART auswendig kennst…)
    Ich schau dann bei Gelegenheit selbst nochmal.

  80. #80 Physik-Fan
    27. September 2016

    @Niels
    @MartinB

    Danke Euch Beiden für die eingehenden Antworten (vor allem #76, whouwww). Es ist klar, bei einer Kräfteformulierung der Grav. wird die RZ-Krümmung obsolet. Beides zusammen macht ja keinen Sinn. Allerdings, bei der rein geometrischen Interpretation ergibt sich das Äquivalenzprinzip praktisch von selbst. Das ist eine ihrer großen Stärken. Dadurch, dass bei den Raumzeitfundamenten selbst angesetzt wird, ist auch die grav. Zeitdilatation kein prinzipielles Problem. Bei Kräften frage ich mich schon, wie dadurch Uhrengänge beeinflusst werden sollen, vor allem in einer so universellen, aber gleichzeitig relativen Weise. Ob das mit Kräften plausibel möglich ist? Vielleicht wäre hier ein Punkt, auf die geometrische Interpretation zurückzugreifen.

    Bewusst habe ich nach der Art der Kräfte gefragt, ob klassisch oder quantentheoretisch. Klassisch bringt es doch wenig, Was soll wieder nur eine klassische Alternative zur ART? Man braucht eine QFT, denn die Kräfteformulierung muss auf eine Quantengravitation hinauslaufen. Auf was sonst?

    Martin, Du hast auf die String-Theorie als mögliches Ziel hingewiesen, aber ist von der überhaupt noch was zu erwarten? Für mich stehen die Chancen eher dafür, zum größten intellektuellen Fehlschlag der Geschichte zu werden. Ein riesiger Aufwand an Arbeitsstunden ist schon hineingeflossen und was ist dabei an Greifbaren herausgekommen? Es gibt nicht eine einzige präzise, überprüfbare, spezifische Vorhersage. Bei den empirischen Indizien sieht’s bis jetzt mau aus. Durchaus mögliche Effekte höherer Dimensionen blieben aus (was Du angesprochen hast). Auch die SUSY wackelt bedenklich. Darüber kommen ja die Fermionen hinein, sonst wäre es nur eine bosonische Theorie. Das SM konnte man bisher nicht reproduzieren. Alle bisher ausgearbeiteten STs sind m.W. nicht konform dazu, es hakt mehr oder minder stark. Klar, in der gigantischen Weite der Calabi-Yau-Landschaft könnte dass SM irgendwo stecken, aber ob es wirklich so ist, weiß man nicht. Niemand konnte bisher erhärten, dass eine Lösung gibt. Auf der anderen Seite ist der enorme modell- und hypothenmäßige Aufbau. Es gibt kein Beispiel in der Physikgeschichte, dass ein so befrachtetes Programm erfolgreich war. Ich sehe da auch prinzipiell fragwürdige Elemente, wie die dimensionale Einbettung. Das ist Mathematik, ist es aber auch physikalisch sinnvoll? Das sollte man mal hinterfragen, anstatt immer weiter zu rechnen.

    Die Kritik von Smolin ist ja allseits bekannt, aber weniger bekannt sein dürfte, dass sogar Lisa Randall (!) die ST als Forschungsprogramm ablehnt. Sie hält es nicht für aussichtsreich. Sie fordert die String-Leute auf, sich um überprüfbare Voraussagen zu bemühen. Statt einem deduktiven Ansatz wie bei ST, favorisiert sie eine iterative Vorgehensweise. Das bedeutet Modelle zu erschaffen, die sich testen lassen und dann, auf Basis der Ergebnisse, die Modelle weiter zu entwickeln. Sie glaubt nicht daran, über einen so riesigen Energiebereich hinweg, direkt die richtige Theorie treffen zu können. Von Argument der mathematischen Eleganz, was die String-Leute gerne anführen, hält sie auch nicht viel, ist für sie trügerisch.

  81. #81 MartinB
    27. September 2016

    @Physik-Fan
    “Bei Kräften frage ich mich schon, wie dadurch Uhrengänge beeinflusst werden sollen, vor allem in einer so universellen, aber gleichzeitig relativen Weise. ”
    Dazu hatte ich ja in der oben verlinkten Serie was geschrieben – ja, das geht.

    ” ist von der überhaupt noch was zu erwarten? ”
    Da traue ich mir keine Antwort zu – ich bin ja auch kein Freund der Stringtheorie, aber sie hat schon ziemlich clevere Ideen hervorgebracht (z.B. holographische Theorien). Trotzdem glaube ich trotz aller Vorteile auch nicht, dass die ST am Ende die Lösung ist. Ich erwähne sie aber gern, weil sie ein Beispiel dafür ist, dass man mit einer flachen Minkowski-Raumzeit arbeiten kann.

  82. #82 Physik-Fan
    27. September 2016

    @Niels
    … ich finde, dass sich reine Philosophen bei Aussagen zur ART, Quantenfeldtheorie oder gar zu Quantengravitationstheorien wie der Stringtheorie viel zu weit aus dem Fenster lehnen. Meiner Ansicht ist die Grundvoraussetzung dafür nämlich, dass man mindestens die dahinter stehende Mathematik hundertprozentig verstanden hat. Und dann ist es immer noch ein weiter Weg dahin zu erfassen, was das Ganze jetzt physikalisch bedeutet. Das schaffen auf diesen Gebieten schon nicht besonders viele Physiker und Mathematiker, selbst nachdem sie sich damit jahrzehntelang auseinandergesetzt haben.
    Bei jemand, der an der Uni nur Philosophie studiert hat, sehe ich da eigentlich keine realistischen Chancen.

    Ich glaube, Du schätzt Intention und Möglichkeiten dieser Leute nicht ganz richtig ein. Das ist spezialisierte Naturphilosophie und Wissenschaftshistorie. Ich komme ja auch nicht aus dieser Ecke und möchte mal eine Lanze dafür brechen. Man beschäftigt sich eingehend mit den Grundfragen, welche sich um die Theorien ranken. Was sagt uns eine Theorie über die Natur? Was bedeuten ihre Konstrukte physikalisch? Ist das Modell konsistent und plausibel, in sich und in Bezug zu anderen Theorien? Auf welchen Annahmen beruht die Theorie? Wie ist die Verträglichkeit mit allg. physikalischen Prinzipien? Ist die Theorie überpüfbar? Stichworte: Ockhamsches Prinzip, Poppersche Falsifizierbarkeit. Das sind wesentliche Gesichtspunkte und die sind nun mal auf einer Metaebene, mathematische Details braucht man nicht dazu

    Die “Wahrheit” liegt sowieso nicht in der Mathematik. Mathematik ist was Abstraktes und für die Physik eine Methode, um zu quantitativen Aussagen zu kommen. Aus der Mathematik alleine heraus kommt man nicht zu Erkenntnissen über die Natur. Es stellen sich immer die Fragen der Interpretation, Relevanz (eine mathematische Lösung muss nicht realisiert sein) und empirischen Verifikation. Ohne das Letztere hängt sowieso alles in der Luft. Gerne wird auch von Theoretikern die mathematische Eleganz, Schönheit, Einfachheit betont. Das ist als Argument trügerisch. Der schönsten Mathematik kann die nüchterne Realität entgegenstehen.

    Für Altmeister wie Heisenberg, Pauli und Einstein war die Metaebene immer wichtig. Sie diskutierten intensiv über die Interpretation der Konstrukte (z.B. die Wellenfunktion) und die Prinzipien für Theorien. Mathematik war für sie ein Mittel zum Zweck (für Einstein gilt das jedenfalls in jüngeren bis mittleren Jahren). Über manchen mokierte sich Pauli, ob er denn “unter die reinen Mathematiker gegangen sei” (später bekam dies auch Einstein zu hören). Vielleicht fehlt einer wie Pauli heute. Man glaubt wohl, sich komplexe Modelle und ganze Sätze von Hypothesen leisten zu können; wenn man nur intensiv genug Mathematik betreibt, wird man schon auf das Richtige kommen. Aber das ist sehr optimistisch, ein gewagtes Spiel. Dermaßen befrachtete Theorien waren kaum jemals erfolgreich. Wenn es dann noch um die experimentelle Verifikation schlecht steht, wird’s vollends abenteuerlich. Musterbeispiel: String-Theorie.

    Aber mathematische Aspekte unbetrachtet zu lassen, wäre auch nicht richtig. Es kommt zum einen darauf an, wie man zu den Ergebnissen gelangt ist und zum anderen kann eine formale Beschreibung als das Einzige erscheinen, was möglich ist (Beispiel: Kopenhagener Interpretation der QM). Oft genug stecken in der mathematischen Herleitung “komische” Sachen drin, wenn man z.B. ad hoc Zick-Zack-Pfade einführt, voreilig verallgemeinert oder von nicht streng bewiesenen Aussagen ausgeht (Beispiel: die angebliche Nichtendlichkeit der Supergravitation). Aber selbst allg. anerkannte Beweise können sich als falsch herausstellen. Berühmtes Beispiel: Der Vollständigkeitsbeweis der QM durch von Neumann. Das Theorem wurde unisono für korrekt betrachtet, bis Bell mal genauer nachprüfte und feststellte, dass gewisse Grundannahmen nicht allg. gültig sind und zwar so klar, dass er von absurd redete. Ein lehrreiches Beispiel! Man sollte sich ggüb. mathematischen Konstrukten/Beweisen/Ableitungen in der Physik eine gewisse Skepsis bewahren.

  83. #83 MartinB
    28. September 2016

    @Physik-Fan
    Aber interessant ist doch, dass alle Namen, die du nennst, Namen von Physikern sind, nicht von Philosophen. Ich denke, genau das ist das, was Nils deutlich machen will: Ohne ein sehr genaues Veständnis der Theorien kann man auch nicht sinnvoll drüber philosophieren.
    Positives Beispiel: Der Artikel zu E=mc², den ich neulich für meinen Artikel verwendet habe, geschrieben von jemandem, der die SRT offensichtlich sehr sauber durchdacht hat.
    Negatives Beispiel: Der Philosophiedozent, der mir Ende der 80er erzählte, die Quantenmechanik sei ja längst widerlegt, denn es gäbe Experimente, die die sogenannte Bellsche Ungleichung verletzen (und der dann sagte, die Physiker würden das aber alle totschweigen, um ihre Theorie zu retten).

  84. #84 Niels
    28. September 2016

    @Physik-Fan

    Es ist klar, bei einer Kräfteformulierung der Grav. wird die RZ-Krümmung obsolet. Beides zusammen macht ja keinen Sinn.

    So einfach ist es aber auch nicht unbedingt.
    Man kann mathematisch zeigen, dass die ART die einfachste vierdimensionale “Raumkrümmungstheorie” ist. Es sind aber auch sehr viel kompliziertere metrische Gravitationstheorien möglich, bei denen zum metrischen Tensor beispielsweise auch noch Skalare oder Vektoren dazukommen.
    Das kann man dann durchaus so interpretieren, dass es sowohl eine Kraft als auch eine Raumzeitkrümmung gibt. Solche Theorien sind deswegen interessant, weil man dann möglicherweise ohne dunkle Materie und/oder dunkle Energie auskommt.

    Auf der anderen Seite ist der enorme modell- und hypothenmäßige Aufbau. Es gibt kein Beispiel in der Physikgeschichte, dass ein so befrachtetes Programm erfolgreich war.
    Das ist Mathematik, ist es aber auch physikalisch sinnvoll?

    Na ja, ich glaube, du unterschätzt die relativistischen Quantenfeldtheorien ein bisschen. Die sind mathematisch auch kein Zuckerschlecken, daran wurde auch sehr lang herumentwickelt und “modell- und hypothenmäßig” kann sich wohl auch fast niemand Elementarteilchen als bestimmte Anregungsmoden von Quantenfeldern vorstellen.

    Auch die ART war zur damaligen Zeit in ihren mathematischen Implikation ziemlich der Hammer. Das ist auch alles ziemlich abgedreht, daran hat man sich nur mittlerweile gewöhnt.
    Das Universum wird dort mathematisch durch eine pseudoriemannsche 4er Mannigfaltigkeit beschrieben, die Geschwindigkeit eines Körpers durch einen 4er-Vektor im Tangentialraum in einem Punkt dieser Mannigfaltigkeit, …

    Dass eine Quantisierung dieser Theorie oder gar eine Vereinheitlichungstheorie aller Kräfte (wie die Stringtheorie) mathematisch sehr anspruchsvoll sind wird halte ich für zweifelsfrei.
    Dass es für unser Realitätsverständnis auf etwas Neuartiges hinausläuft muss kommt mir ebenfalls sehr wahrscheinlich vor.

    Statt einem deduktiven Ansatz wie bei ST, favorisiert sie eine iterative Vorgehensweise. Das bedeutet Modelle zu erschaffen, die sich testen lassen und dann, auf Basis der Ergebnisse, die Modelle weiter zu entwickeln. Sie glaubt nicht daran, über einen so riesigen Energiebereich hinweg, direkt die richtige Theorie treffen zu können.

    Ist ja eine nette Idee, aber das will nun einmal nicht klappen.
    Der LHC hat nun mal keine neuartige Physik geliefert. Auch die energiereichsten, milliardenjährigen Photonen zeigen im Rahmen unserer Messgenauigkeit keine Laufzeitunterschiede zu energiearmen Photonen.
    usw.
    Wenn unsere bisherigen Theorien enorm gute Näherungen für den uns zugänglichen Energiebereich sind, wonach es immer mehr aussieht, dann kann eine iterative Vorgehensweise eben auch nicht weiterhelfen.

    Man beschäftigt sich eingehend mit den Grundfragen, welche sich um die Theorien ranken. Was sagt uns eine Theorie über die Natur? Was bedeuten ihre Konstrukte physikalisch? Ist das Modell konsistent und plausibel, in sich und in Bezug zu anderen Theorien? Auf welchen Annahmen beruht die Theorie? Wie ist die Verträglichkeit mit allg. physikalischen Prinzipien? Ist die Theorie überpüfbar? Stichworte: Ockhamsches Prinzip, Poppersche Falsifizierbarkeit. Das sind wesentliche Gesichtspunkte und die sind nun mal auf einer Metaebene, mathematische Details braucht man nicht dazu

    Da bin ich absolut anderer Meinung.
    Ohne Verständnis der Mathematik hat man doch überhaupt keine Chance, diese Fragen zu beantworten.
    Du bringst mit den Bellschen Ungleichung doch selbst ein sehr gutes Argument für diesen Umstand. Diese kann man ohne gute Kennt­nis der Mathematik nicht einmal verstehen, geschweige denn jemals formulieren. Einen experimentellen Test dazu entwickeln ebenfalls nicht.

    Man glaubt wohl, sich komplexe Modelle und ganze Sätze von Hypothesen leisten zu können; wenn man nur intensiv genug Mathematik betreibt, wird man schon auf das Richtige kommen.

    Eigentlich ist es doch so, dass man momentan einfach keine andere Möglichkeit mehr hat.
    Die Theoretiker würden sich durchaus mit mathematisch simplen, konzeptionell einfachen Theorien anfreunden. Die hat man allerdings schon durchprobiert, so klappt es einfach nicht.

    Der Vollständigkeitsbeweis der QM durch von Neumann. Das Theorem wurde unisono für korrekt betrachtet, bis Bell mal genauer nachprüfte und feststellte, dass gewisse Grundannahmen nicht allg. gültig sind und zwar so klar, dass er von absurd redete. Ein lehrreiches Beispiel!

    Tatsächlich?
    Die Theorie der Führungswellen hat de Broglie doch schon 1926 entwickelt.
    Gut, das mag nicht so wahnsinnig bekannt gewesen sein, aber die Bohmsche Mechanik war doch schon vor Bells Arbeiten bekannt.
    Das bewies doch, dass von Neumanns Theorem falsch war?

  85. #85 Physik-Fan
    28. September 2016

    @MartinB #75

    Ich habe mal Deine Artikel durchgelesen. Von der Feynmanschen Grav.theorie wusste ich, aber nichts Näheres darüber. Daher kommt es also, man hat diese exotische Formulierung hervorgekramt (die Feynman selbst nicht so ernst nahm und mehr als schöne Übung betrachtete). Ganz geistig verdaut habe ich den Stoff noch nicht, vielleicht weil ich in den herkömmlichen Vorstellungen von Kräften (klassisch, quantentheoretisch) und RZ-Krümmung stecke. Deine Betrachtung mit der Energie, an die die Grav. koppelt und so in allen Feldern drin steckt, ist gut. Vielleicht ist das der Schlüssel.

    Zunächst möchte ich mal betonen, dass mir Deine Herangehensweise an Physik sehr gefällt, also die Dinge in Bildern auszudrücken und Zusammenhänge auf den Punkt zu bringen, in wenigen, klaren Sätzen. Genauso kommt man zu einem tieferen physikalischen Verständnis. Das ist auch meine Meinung. Wenn jemand an einer Theorie brütet, hat er typischerweise auch Bilder und ein paar Ideen im Kopf (klar, es gibt andere Fälle).

    Dein Bild im Teil 3 mit dem Gummituch kann zu Missverständen führen, denn es suggeriert ein Medium. Aber, sowie ich es verstanden habe, in der Feldformulierung gibt es das Medium als dynamisches Element nicht. Das Feld wirkt direkt auf Körper und Uhrengänge und verzerrt sie. Da ergibt sich sofort die Frage, was denn das für eine Kraft ist. Wie wechselwirkt wie? Wie schafft es die Kraft, gegen die atomaren Kräfte anzukommen und das unter allen Bedingungen und unabhängig von der Art der Bindungskräfte? Wie verändert die Kraft die zeitlichen Abläufe? Das ist noch seltsamer. Man braucht eine Theorie der Kraft selbst (am besten QF-theoretisch), die das alles beantwortet, sonst ist es nur ein Formalismus, der sich auch nur an den Phänomenen orientiert.

    Verglichen damit sieht für mich die RZ-Formulierung plausibler und modellmäßig klarer, sauberer strukturiert aus (jedenfalls nach meinem aktuellen Kenntnis-/Verständnisgrad der Feldformulierung). Da hat man die Materie so wie sie ist, mit den ihr innewohnenden Kräften und dazu das Medium, in dem sie sich befindet. Es ist das Medium, das sich ändert und nur das bestimmt ist die RZ-Bahn. So ergibt sich automatisch die Universalität der Grav.phänomene, darunter das Äquivalenzprinzip und zwar modellmäßig sehr einfach. Eine Kräfteformulierung muss das Gleiche leisten, also ohne eine Zusatzannahme, sonst wäre es ein entscheidender Nachteil.

    Natürlich lässt auch die ART Einiges offen, z.B. der Mechanismus, mit dem Masse/Energie die RZ krümmt. Die ART ist eben auch nur phänomenologisch. Die RZ kann sich beliebig dehnen und die Krümmung unendlich werden, was kaum plausibel ist. Außerdem stellen sich Fragen wie nach dem Urknall selbst, wie/warum er erfolgte, ob es der absolute Anfang oder nur ein Durchgang war. Für das alles braucht man wohl eine Quantengravitation mit einem Hintergrundmodell der RZ.

    Warum die Krümmung unterschiedl. ist, ist gar nicht so “seltsam”, sondern ergibt sich ganz plausibel. Die RZ ist wie ein Stoff. Ein Körper verformt sie zunächst dort, wo der Körper ist und außerhalb davon setzt sich die Verformung durch ihre Spannungseigenschaften fort. Da passt das Gummituchbild gut. Und wie das Gummituch, verformt sich die RZ in ihrer speziellen Weise. Beschrieben wird das durch die Feldgleichung, aber wo kommt sie her? Cartan konnte sie herleiten aus der Erhaltung der Impenergie in einem geschlossenen RZ-Gebiet. Deswegen krümmt sich also die RZ so wie sie es tut. Und warum ist die Impenergie erhalten? Die gleiche Frage stellt sich auch für die Energie. Nöther hat die Energieerhaltung zurückgeführt auf eine Symmetrieeigenschaft der Natur. Analoges bietet sich für die Impenergie an. So kann die ART komplett aus fundamentalen Naturprinzipien abgeleitet werden. Ob eine Feldformulierung das leisten kann?

    Außerdem, auch nicht unwichtig, die ART beschreibt ja keine äußere Krümmung, sondern eine innere in Form einer Riemannschen Mannigfaltigkeit. Besser wäre es etwa von RZ-“Verzerrung” zu reden. Der 4-D-Raum kann flach sein, also nicht anders wie in der Feldformulierung. Er ist nur in sich verzerrt. Warum braucht man dann eine Feldformulierung? Mit der Metrik geht es doch einfacher und m.E. plausibler.

  86. #86 MartinB
    28. September 2016

    @Physik-Fan
    “man hat diese exotische Formulierung hervorgekramt (die Feynman selbst nicht so ernst nahm und mehr als schöne Übung betrachtete). ”
    ??? Reden wir über dasselbe? Du meinst nicht die Geschichte mit der heißen Platte, oder? (Die ist nur ne Veranschaulichung, richtig, die mir als Ausgangspunkt dient.) Mir geht es um die Formulierung als Wechselwirkung mit einem (Spin-2)-Feld im 3. Teil, vor allem am Ende, wo die Gleichungen sind:
    https://scienceblogs.de/hier-wohnen-drachen/2015/03/21/ist-die-raumzeit-gekruemmt-teil-iii-die-zeit/?all=1
    Das ist ganz sicher keine “schöne Übung”, sondern fundamentale Physik.

    “Dein Bild im Teil 3 mit dem Gummituch kann zu Missverständen führen, denn es suggeriert ein Medium.”
    Jein. Letztlich ist das Vakuum in der QFt das Medium, weil es Anregungen von Quantenfeldern zulässt. Und abgesehen von der Quantisierung ist die Analogie Gummituch – Quantenfeld perfekt, weil beide dieselbe Lagrangedichte haben (abgesehen davon, dass man die Größen darin unterschiedlich interpretieren muss.)

    “Da passt das Gummituchbild gut”
    Ne, für die Verformung der Raumzeit passt das Gummituch eher nicht so gut, weil sich im Gummituch immer nur der Raum verformt und nicht die zeit und weil sich die Raumzeit auch nicht wirklich wie ein elastisches Medium verhält.

    “Warum braucht man dann eine Feldformulierung? Mit der Metrik geht es doch einfacher und m.E. plausibler.”
    Was “plausibel” ist, ist immer eine Frage des Standpunkts – solange zwei Formulierungen einer Theorie dieselben vorhersagen machen, sind sie gleich gut. Ist Newton besser/plausibler als Lagrange oder Hamilton? Alle Formulierungen haben ihre Vorteile – Hamilton hilft, wenn man zur kanonischen Quantisierung geht, Lagrange, wenn man Pfadintegrale macht. Und die Spin-2-Formulierung hilft, wenn an über Quantengravitation nachdenkt, siehe die feynman Lectures on Gravitation.

    “So kann die ART komplett aus fundamentalen Naturprinzipien abgeleitet werden.”
    Kannst du mir dafür mal nen Link schicken, wo du das genau her hast, das ist mir so nicht klar?

  87. #87 Niels
    29. September 2016

    @Physik-Fan

    Außerdem, auch nicht unwichtig, die ART beschreibt ja keine äußere Krümmung, sondern eine innere in Form einer Riemannschen Mannigfaltigkeit. Besser wäre es etwa von RZ-“Verzerrung” zu reden.

    Na ja, der mathematische Fachbegriff ist eben Krümmung.

    Warum braucht man dann eine Feldformulierung? Mit der Metrik geht es doch einfacher und m.E. plausibler.

    Warum braucht man eine Krümmungsformulierung? 😉
    Was erklärt sie, dass die Feldformulierung nicht leistet?
    Der Unterschied zwischen Krümmung und Feld ist ja gerade, als was man die Metrik physikalisch interpretiert.

    Wie MartinB schon ausgeführt hat:
    Krümmungsbild und Feldbild sind keine unterschiedlichen Theorien, sondern nur verschiedene physikalische Interpretationen der zugrunde liegenden Mathematik.
    Beide machen exakt die selben Vorhersagen, experimentell kann zwischen diesen Interpretationen nicht unterschieden werden.
    (Wobei ich glaube, dass globale Eigenschaften wie die Topologie des Universums oder Extremfälle wie Singularitäten besser im Krümmungsbild funktionieren.
    Vermutlich klappt das aber über irgendwelche Tricks auch im Feldbild.
    Experimente dazu sind jedenfalls nicht ganz so einfach.;-) )

    Das funktioniert also ganz analog zu den verschiedenen Interpretationen der Quantenmechanik.

    In der “Bibel” der ART, nämlich “Gravitation” von Charles W. Misner, Kip S. Thorne und John Archibald Wheeler (Abkürzung MTW), findet man übrigens eine Tabelle (18.1), die Krümmungs-ART und Spin-2-Feldtheorie-ART vergleicht.
    Auch Weinberg verwendet in “Gravitation and Cosmology” den feldtheoretischen Zugang.

    Beschrieben wird das durch die Feldgleichung, aber wo kommt sie her? Cartan konnte sie herleiten aus der Erhaltung der Impenergie in einem geschlossenen RZ-Gebiet.

    Quelle?
    Meines Wissens kann man die Feldgleichungen entweder wie Einstein “herleiten”, also mit Hilfe des Äquivalenzprinzips und Plausibilitätsannahmen.
    Oder mathematisch korrekt, in dem man die Einstein-Hilbert-Wirkung postuliert und das Prinzip der stationären Wirkung anwendet.

    Und warum ist die Impenergie erhalten? Die gleiche Frage stellt sich auch für die Energie. Nöther hat die Energieerhaltung zurückgeführt auf eine Symmetrieeigenschaft der Natur. Analoges bietet sich für die Impenergie an. So kann die ART komplett aus fundamentalen Naturprinzipien abgeleitet werden. Ob eine Feldformulierung das leisten kann?

    Zum einen müsste das dann selbstverstärdnlich auch im Feldbild klappen, es beruht wie gesagt ja auf der selben Mathematik.

    Zum anderen bin ich mir aber sicher, dass das so nicht stimmen kann.

    Die Existenz der Energieerhaltung folgert man üblicherweise aus dem Noether-Theorem. Das klappt in der ART aber nicht so richtig. Das liegt daran, dass die aus Symmetrien über das Noether-Theorem abgeleitete Energiedichte ausschließlich die an das “Gravitationsfeld” koppelnde Materie und Felder beinhaltet und eben gerade nicht das “Gravitationsfeld” selbst.
    Die Ursache bzw. das Hauptargument für die Existenz einer erhaltenen Energie fällt also weg.

    Für unser Universum gilt offensichtlich keine Zeitinvarianz (Homogenität der Zeit), da sich der Skalenfaktor und die Krümmung mit der Zeit ändern. Laut dem Noether-Theorem folgt aber gerade aus dieser Symmetrie die Energieerhaltung.

    In der ART “verallgemeinert” man das Noether-Theorem mit Hilfe sogenannter Killing-Vektorfelder. Killing-Vektorfelder beschreiben Symmetrieeigenschaften bzw. Isometrien der Raumzeit.
    Wenn eine Raumzeit ein oder mehrere Killing-Vektorfelder besitzt, gibt es zu jedem dieser Killing-Vektorfelder jeweils eine Erhaltungsgröße der Raumzeit.

    Es zeigt sich, dass es nur in sogenannten stationären Raumzeiten, also “zeitunabhängigen” Raumzeiten, deren Metrikkoeffizienten keine Funktionen der Zeit enthalten, eine Erhaltungsgröße “Energie” geben kann.
    Die Schwarzschild-Metrik und die verschiedenen Metriken für rotierende und/oder geladene schwarze Löcher sind bekannte Beispiele.
    Die Schwarzschild-Lösung ist zusätzlich auch noch statisch, d.h. es treten keine Kreuzterme wie dr*dt auf. Sie ist also zeitsymmetrisch und zeittranslationsinvariant.

    (Nach mathematischer Definition ist eine Raumzeit genau dann stationär, wenn sie ein zeitartiges Killingvektorfeld besitzt.
    Eine Raumzeit ist genau dann statisch, wenn sie ein hyperflächenorthogonales zeitartiges Killingvektorfeld besitzt.)

    Raumzeiten müssen nicht unbedingt expandieren, um nicht-stationär zu sein. Eine Raumzeit mit mehreren Massen ist zum Beispiel in aller Regel nicht-stationär.

    Sogar für Stationäre Raumzeiten ist die Sache mit der Energie aber gar nicht so einfach.
    Stationäre Raumzeiten besitzen per Definition ein zeitartiges Killingvektorfeld. Zu solchen Killingvektorfeldern gehört dann als Erhaltungsgröße die “Energie”. Das ist wegen der “Graviationsenergie” aber ein verdammt kompliziertes Thema und nicht wirklich das, was man sich sonst unter Energie vorstellt.
    Da ist das Stichwort dann die sogenannte Komar-Masse.

    Für allgemeine Raumzeiten ist es bisher noch nicht gelungen, eine überzeugende Definition für eine erhaltene Energie in der ART zu finden, obwohl daran seit Jahrzehnten geforscht wird.
    Bisher ist nicht geklärt, ob das eine grundlegende Eigenschaft ist oder ob man mit Hilfe einer Erweiterung der ART, also z.B. einer Quantengravitation, so eine erhaltene Energie nicht doch definieren könnte.
    Das Problem ist, dass es bisher noch niemandem gelungen ist, der Raumzeit im Rahmen der ART sinnvoll eine Energie(dichte) zuzuordnen.
    Wenn also “Energie an die Raumzeit verloren geht”, kann man keine Aussage darüber machen, ob da etwas erhalten bleibt oder nicht.
    (Es gibt zwar Versuche für eine Definition mit Hilfe von Pseudotensoren, das hat dann aber wenig mit dem zu tun, was man üblicherweise unter Energie versteht. Oder unter Erhaltungsgröße. Außerdem gibt es verschiedene Ansätze, wobei keiner Ansatz bisher allgemein als besonders nützlich akzeptiert wurde.)

    Selbst wenn doch noch ein Weg gefunden wird: Es ist mittlerweile sicher, dass das dann nur noch wenig mit dem zu tun haben wird, was wir momentan unter dem Begriff Energie verstehen.

    Cartan starb 1951. Wenn seine Impenergie (hab ich vorher noch nie gehört) das von dir behauptete leisten würde, wäre dieses Problem also seit über 60 Jahren gelöst.
    Ich kann mir nicht vorstellen, dass das komplett übersehen wurde.

  88. #88 MartinB
    29. September 2016

    @Niels
    Danke für die Erläuterungen zum Killing-Vektorfeld.

    Das mit der Impenergie und Cartan kommt möglicherweise hierher:
    https://www.spiegel.de/forum/wissenschaft/gravitationswellen-nie-ein-mensch-zuvor-gemessen-hat-thread-418965-12.html
    Wär mir aber auch völlig neu, dass man das so herleiten kann – sonst würde das ja in neueren ART-Büchern auch mal drinstehen…

  89. #89 Bullet
    29. September 2016

    @Martin:

    Negatives Beispiel: Der Philosophiedozent, der mir Ende der 80er erzählte, die Quantenmechanik sei ja längst widerlegt, denn es gäbe Experimente, die die sogenannte Bellsche Ungleichung verletzen (und der dann sagte, die Physiker würden das aber alle totschweigen, um ihre Theorie zu retten).

    Hab ich jetzt nicht so ganz begriffen. Das wird doch gar nicht totgeschwiegen.

  90. #90 MartinB
    29. September 2016

    @Bullet
    Der Typ hatte es so verstanden:
    Die bellsche Ungleichung ist verletzt, wenn die verletzt wird, ist das ganz doll schlimm und zeigt, dass die QM nicht gilt, aber keiner redet drüber.
    Ja, so ahnungslos kann man sein und als Dozent Philo-Seminare leiten. Kurz danach habe ich dann das Nebenfach gewechselt…

  91. #91 Bullet
    29. September 2016

    Ah. Daß man als Philo-Dozent mal darüber nachdenken können müßte, ob die Bell-Ungleichung überhaupt statthaft ist (also Reali- und Lokalität überhaupt sein müssen), ist wahrscheinlich zu hoch gegriffen, wa? *facepalm*

  92. #92 Physik-Fan
    29. September 2016

    @Niels
    Man kann mathematisch zeigen, dass die ART die einfachste vierdimensionale “Raumkrümmungstheorie” ist. Es sind aber auch sehr viel kompliziertere metrische Gravitationstheorien möglich, bei denen zum metrischen Tensor beispielsweise auch noch Skalare oder Vektoren dazukommen.
    Das kann man dann durchaus so interpretieren, dass es sowohl eine Kraft als auch eine Raumzeitkrümmung gibt. Solche Theorien sind deswegen interessant, weil man dann möglicherweise ohne dunkle Materie und/oder dunkle Energie auskommt.

    An Mischlösungen habe ich auch schon gedacht und in #80 erwähnt, in Zusammenhang mit der grav. Zeitdilatation. Zeitdilatation ist doch so fundamental, wie soll eine Kraft das bewerkstelligen? Eine solche Wirkung hätte ich gerne mal gesehen. Vielleicht geht’s plausibler über die Metrik.

    Na ja, ich glaube, du unterschätzt die relativistischen Quantenfeldtheorien ein bisschen. Die sind mathematisch auch kein Zuckerschlecken, daran wurde auch sehr lang herumentwickelt und “modell- und hypothenmäßig” kann sich wohl auch fast niemand Elementarteilchen als bestimmte Anregungsmoden von Quantenfeldern vorstellen.

    Das war auch ein langer Entwicklungsprozess und in Korrespondenz mit den Experimenten. Dabei waren die guten Übereinstimmungen ein starkes Argument für die QED und ihre Nachfolger, sonst hätte die QFT wohl nicht ihre überragende Stellung bekommen. Es gibt auch anschauliche Elemente in der QFT. Die Feynman-Diagramme sind nichts anderes. Klar, man muss wissen, was die genau bedeuten, dass man sie nicht 1:1 als physikalischen Prozess verstehen kann. Aber so hat man wenigstens Anschauungshilfen.

    Der LHC hat nun mal keine neuartige Physik geliefert. Auch die energiereichsten, milliardenjährigen Photonen zeigen im Rahmen unserer Messgenauigkeit keine Laufzeitunterschiede zu energiearmen Photonen.
    usw.
    Wenn unsere bisherigen Theorien enorm gute Näherungen für den uns zugänglichen Energiebereich sind, wonach es immer mehr aussieht, dann kann eine iterative Vorgehensweise eben auch nicht weiterhelfen.

    Abwarten, die Experimente mit 13 TeV laufen erst seit Mai. Der LHC wird auch kontinuierlich verbessert, die Luminosität gesteigert, bei der Energie geht noch was. Erst in ein paar Jahren wird das Leistungspotenzial ausgeschöpft sein. Wenn sich dann hartnäckig nichts Neues zeigt, dann steckt die Teilchenphysik wirklich in der Tinte. Die deduktiven Theorien jenseits der Überprüfbarkeit werden dann auch nicht groß weiterhelfen. Sie bleiben abstrakte Konstruktionen mit unbekanntem Realitätsgehalt. Bei den induktiven wird man immerhin gewisse Aufschlüsse für die weitere Entwicklung gewonnen haben, denn keine Effekte sind auch eine Aussage. Dann wären wieder die Experimentatoren gefordert und die sind ja auch recht einfallsreich.

    Ohne Verständnis der Mathematik hat man doch überhaupt keine Chance, diese Fragen zu beantworten.

    Ich habe doch explizit versucht, Dinge anzusprechen die eben nicht den mathematischen Apparat betreffen, wie Modell, Hypothesen, Ergebnisse, halt das, was die darin steckende Physik ausmacht.

    Du bringst mit den Bellschen Ungleichung doch selbst ein sehr gutes Argument für diesen Umstand. Diese kann man ohne gute Kenntnis der Mathematik nicht einmal verstehen, geschweige denn jemals formulieren. Einen experimentellen Test dazu entwickeln ebenfalls nicht.

    Zeilinger hat es mal recht schön auseinander gedröselt. Der Bellschen Ungleichung liegt ganz simple Mengenlogik zugrunde, nämlich dass eine Teilmenge gleich oder weniger Elemente wie die Obermenge hat. Das stellt man für verschiedene Eigenschaften der Objekte auf und mixt es. So kommt es zur Ungleichung. Die wendet man dann auf die EPR-Situation an, d.h. mit voneinander entfernten Objekten. Dabei stecken implizit die Prinzipien der klassischen Physik (genauer: spezialrelativ. Physik) drin, wie objektive Realität und lokale Kausalität. Dass die QM eine die Ungleichung verletzende Aussage liefert, muss man natürlich ausrechnen, aber die Ungleichung selbst ist eher trivial. Natürlich man muss auf das Ganze erst mal kommen, das war genial.

    Eigentlich ist es doch so, dass man momentan einfach keine andere Möglichkeit mehr hat.
    Die Theoretiker würden sich durchaus mit mathematisch simplen, konzeptionell einfachen Theorien anfreunden. Die hat man allerdings schon durchprobiert, so klappt es einfach nicht.

    Kann man das wirklich so einfach sagen? Erstens kommt es auf den Ideenreichtum an und zweitens auf den Aufwand, mit dem man die Ideen verfolgt. Im Letzteren liegt vielleicht ein großes Problem. Smolin hat vor ca. 10 Jahren kritisiert, dass zu viel an der ST gearbeitet wird, dass sie zu stark die theoretische Szene dominiert. Das war ja der Kern seiner Kritik, weniger die ST selbst. In der Kosmologie ist man auch festgefahren. Vielleicht muss die theoretische Physik offener werden. Und dann braucht man dringend experimentelle Befunde. In der Spektrum war jetzt z.B. ein Artikel über neue Experimente, um eventuellen Quanteneigenschaften der Grav. auf die Spur kommen.

    Die Theorie der Führungswellen hat de Broglie doch schon 1926 entwickelt.
    Gut, das mag nicht so wahnsinnig bekannt gewesen sein, aber die Bohmsche Mechanik war doch schon vor Bells Arbeiten bekannt.
    Das bewies doch, dass von Neumanns Theorem falsch war?

    Bei de Broglie war es die Materiewelle. Die wurde dann durch die Wellenfunktion von Schrödinger und die Kopenhagener Interpretation abgelöst. Deren Vertreter betrachteten die QM bereits als vollständig, was dann durch v. Neumann bewiesen schien. Bei Bohm ist mir nicht bekannt, dass er dem Widerspruch zu Neumann nachging, noch dass es ein anderer wegen seiner Theorie getan hat. Sie hat auch keinen großen Anklang gefunden. Empirisch bringt sie nichts Neues und mit der Nichtlokalität des Bohmschen Feldes ist sie als realistische Formulierung sehr fragwürdig. Es wird quasi nur der Teufel durch den Beelzebub ersetzt. Aber vielleicht hat’s Bell angeregt. Bohm war ja auch derjenige, der das EPR-Experiment mit dem Spin formulierte.

  93. #93 MartinB
    29. September 2016

    @Bullet
    ” ob die Bell-Ungleichung überhaupt statthaft ist (also Reali- und Lokalität überhaupt sein müssen), ist wahrscheinlich zu hoch gegriffen, wa?”
    Scheint so.
    Später lernte ich dann in dem Seminar, dass Poincares Wissenschaftstheorie sagt, dass man Theorien aufstellt und die dann durch Experimente prüft. (Wer hätte es gedacht, und der gute Poincare hatte ein bisschen mehr zusagen als das, IIRC).

  94. #94 MartinB
    29. September 2016

    @Physikfan
    “Zeitdilatation ist doch so fundamental, wie soll eine Kraft das bewerkstelligen? Eine solche Wirkung hätte ich gerne mal gesehen. ”
    Ich habe doch die Gleichungen quasi explizit in dem 3. teil des oben verlinkten textes reingeschrieben!? Man muss wirklich nur g_munu als nen Feld betrachten, das an Energien koppelt; Energieterme in der QFT sind immer zweite Ableitungen nach der Zeit, fertig ist der Zeiteinfluss.

    “Die deduktiven Theorien jenseits der Überprüfbarkeit werden dann auch nicht groß weiterhelfen. Sie bleiben abstrakte Konstruktionen mit unbekanntem Realitätsgehalt.”
    Naja, Einstein hatte für die ART zunächst auch keinerlei experimentelle Anhaltspunkte – die Theorie wurde rein deduktiv entwickelt, und erst dann hat Einstein mal nach Dingen wie Periheldrehung geguckt.

  95. #95 Physik-Fan
    29. September 2016

    @MartinB
    Das mit der Impenergie und Cartan kommt möglicherweise hierher:
    https://www.spiegel.de/forum/wissenschaft/gravitationswellen-nie-ein-mensch-zuvor-gemessen-hat-thread-418965-12.html
    Wär mir aber auch völlig neu, dass man das so herleiten kann – sonst würde das ja in neueren ART-Büchern auch mal drinstehen…

    Ich habe es aus “Gravitation und Raumzeit” von J.A. Wheeler. Zusammen mit “Physik der Raumzeit” (auch von Wheeler), wo es um die SRT geht, sind das die besten Einführungen in die RT, die ich kenne. Ich kann mir das gut z.B. auch als Vorlesungsvorbereitung vorstellen. Der physikalische Gehalt der Theorien kommt gut rüber. Mathematisch sollte man nicht zu viel erwarten. Was drin ist, ist sehr umständlich hingeschrieben, aber gut, es soll prinzipiell für jedermann verständlich sein.

    Ich mal kurz die Kapitel 6 u. 7 überflogen. Es geht topologisch noch tiefer, nämlich mit Betrachtungen über den orientierten Rand von Mannigfaltigkeiten. Angewendet auf die ART sind das die begrenzenden Würfel eines RZ-Blocks. So kommt man auf die Erhaltung der Impenergie. An einer Stelle heißt es, dass Cartan mit diesen topologischen Betrachtungen zu Einsteins Theorie gelangte (was sicher die Feldgleichung bedeutet). An anderen Stellen ist allerdings nicht so klar, was aus was folgt. Vielleicht habe ich’s auch überinterpretiert. Muss ich mir etwas genauer ansehen.

  96. #96 MartinB
    29. September 2016

    @Physik-Fan
    “Vielleicht habe ich’s auch überinterpretiert.”
    Das scheint mir sehr wahrscheinlich…

  97. #97 Niels
    29. September 2016

    @MartinB @Physik-Fan

    Wär mir aber auch völlig neu, dass man das so herleiten kann – sonst würde das ja in neueren ART-Büchern auch mal drinstehen…

    Damit gelangt man auf verblüffend direkte und elegante Weise zur Feldgleichung. Der Mann, dem dieses Sensationelle gelang, ist Elie Cartan, ein französischer Mathematiker, auch so ein Genie, und es war um 1929.

    Wenn das schon 1929 hergeleitet wurde, müsste das nicht nur in den neueren Büchern stehen.
    Ich kenne die populärwissenschaftlichen Bücher von Wheeler nicht, es ist aber schon seltsam, dass es dieser Aspekt nicht in das von ihm mitverfasste Lehrbuch, nämlich den MTW, geschafft hat. Und bei dessen 1300 Seiten lag es bestimmt nicht daran, dass es wegen Platzmangel weggelassen werden musste.

    @Physik-Fan

    Es geht topologisch noch tiefer, nämlich mit Betrachtungen über den orientierten Rand von Mannigfaltigkeiten. Angewendet auf die ART sind das die begrenzenden Würfel eines RZ-Blocks.

    Hört sich für mich spontan nicht wie Topologie, sondern wie der stokessche Integralsatz an…

    Eine solche Wirkung hätte ich gerne mal gesehen.

    Na ja, eben die Einstein-Hilbert-Wirkung?

    Es gibt auch anschauliche Elemente in der QFT. Die Feynman-Diagramme sind nichts anderes. Klar, man muss wissen, was die genau bedeuten, dass man sie nicht 1:1 als physikalischen Prozess verstehen kann. Aber so hat man wenigstens Anschauungshilfen.

    Eigentlich sind Feynman-Diagramme bildliche Kurzdarstellungen für mathematische Formeln.
    Das erste Diagramm der Potenzreihe isoliert zu betrachten und so zu tun, als ob die tatsächliche Wechselwirkung nur darauf beruht, dass klar definierte, durch den Raum fliegende Punktteilchen virtuelle Eichbosonen austauschen, ist eigentlich so falsch, dass das als Anschauungshilfe meiner Meinung nach eher schädlich ist.

    Ich habe doch explizit versucht, Dinge anzusprechen die eben nicht den mathematischen Apparat betreffen, wie Modell, Hypothesen, Ergebnisse, halt das, was die darin steckende Physik ausmacht.

    Ich bezweifle eben einfach, dass es bei physikalischen Theorien Dinge gibt, für die die Mathematik keine Rolle spielt. Die Sprache der Physik ist nun mal die Mathematik.
    Aber vielleicht hilft es weiter, wenn du mal ein paar konkrete Beispiele nennst?

    Dass die QM eine die Ungleichung verletzende Aussage liefert, muss man natürlich ausrechnen, aber die Ungleichung selbst ist eher trivial. Natürlich man muss auf das Ganze erst mal kommen, das war genial.

    Mag ja sein, aber glaubst du wirklich, dass genügend Naturphilosophen und Wissenschaftshistoriker irgendwann die Ungleichungen aufgestellt und experimentell überprüft hätten, wenn man ihnen genügend Zeit gegeben hätte?
    Meiner Ansicht bestünde nur dann eine Chance dafür, wenn die sich mal hingesetzt und gründlich die Mathematik der QM durchgeackert hätten.

    Kann man das wirklich so einfach sagen? Erstens kommt es auf den Ideenreichtum an und zweitens auf den Aufwand, mit dem man die Ideen verfolgt.

    Man kann natürlich nie belegen, dass es prinzipiell keine mathematisch simple, konzeptionell einfache Theorie von Allem gibt, die bisher nur niemand gefunden hat.
    Es ist aber natürlich auch nicht so, dass sich alle Theoretiker seit 30 Jahren ausschließlich mit Stringtheorie befassen.

    Vielleicht muss die theoretische Physik offener werden.

    Wenn man sich mal aktuelle Veröffentlichungen anschaut, könnte man eher den Schluss ziehen, da wäre man zu offen.
    Was mittlerweile alles an Teilchen, Kräften, Dimensionen oder gar ganzen Theorien aus dem Hut geschüttelt wird, ist teilweise nicht mehr feierlich.
    Als die 750 GeV Anomalie am LHC bekannt wurde, wurden schon am nächsten Tag neue Theorien auf arXiv hochgeladen. Ohne Genaueres als nur diesen Energiewert zu wissen.
    Da hätte man ruhig etwas geduldiger sein können.

    Bei de Broglie war es die Materiewelle. Die wurde dann durch die Wellenfunktion von Schrödinger und die Kopenhagener Interpretation abgelöst. .

    De Broglie hat die bohmsche Mechanik 1926 entwickelt, Bohm dann noch einmal unabhängig in den 1950ern. Mit der Materiewelle hat das nichts zu tun.
    Um de Broglies Leistung anzuerkennen, wird die Bohmsche Mechanik deswegen auch gerne De-Broglie-Bohm-Theorie genannt.

    Deren Vertreter betrachteten die QM bereits als vollständig, was dann durch v. Neumann bewiesen schien. Bei Bohm ist mir nicht bekannt, dass er dem Widerspruch zu Neumann nachging, noch dass es ein anderer wegen seiner Theorie getan hat.

    Was meinst du mit “vollständig”?

    Soweit ich es verstehe dachte man, Neumanns Beweis zeige, dass es keine Interpretationen der QM mit verborgenen Parametern geben könne.
    Die Bohmsche Mechanik ist aber gerade eine Interpretation mit verborgenen Parametern.

    Damit liege ich wohl auch nicht so falsch. Ich habe mal gegoogelt, in der englischen Wiki findet man:
    Bohm’s original aim was not to make a serious counterproposal but simply to demonstrate that hidden-variable theories are indeed possible.[24] (It thus provided a supposed counterexample to the famous proof by John von Neumann that was generally believed to demonstrate that no deterministic theory reproducing the statistical predictions of quantum mechanics is possible.)

    Empirisch bringt sie nichts Neues und mit der Nichtlokalität des Bohmschen Feldes ist sie als realistische Formulierung sehr fragwürdig.

    Natürlich bringt sie empirisch nichts Neues. Es handelt sich ja nur um eine andere Interpretation der der QM zugrunde liegenden Mathematik.
    Und natürlich ist sie nichtlokal. Das müssen realistische Interpretationen als Folge der Verletzung der Bellschen Ungleichungen schließlich zwangsläufig erfüllen.

  98. #98 MartinB
    29. September 2016

    @Niels
    “ist eigentlich so falsch, dass das als Anschauungshilfe meiner Meinung nach eher schädlich ist.”
    Amen:
    https://scienceblogs.de/hier-wohnen-drachen/2013/08/25/verfuhrerisch-einfach-feynmandiagramme/

    “Natürlich bringt sie empirisch nichts Neues.”
    Wobei aber eine Umformulierung einer Theorie durchaus nützlich sein kann, um neue Ideen für weitere Theorien zu gewinnen, siehe das Beispiel Lagrange/Hamilton/Newton. Insofern finde ich es schon gut und wichtig, immer mehrere Interpretationen zu haben, wenn es sie gibt. (Aber da stimmst du ja vermutlich sowieso überein.)

  99. #99 Niels
    3. Oktober 2016

    @MartinB
    Klar stimme ich da überein.

    Die Sache mit den Feynman-Diagrammen ärgert mich schon seit der zugehörigen Experimentalphysikvorlesung im Studium. Das Problem ist wohl, dass es keine richtigere Veranschaulichung auf diesem Niveau gibt, oder?

  100. #100 MartinB
    3. Oktober 2016

    @Niels
    “Das Problem ist wohl, dass es keine richtigere Veranschaulichung auf diesem Niveau gibt, oder?”
    Ein bisschen sowas habe ich ja gerade mit dem Higgsfeld versucht, aber ja, ne richtig gute Veranschaulichung für Quantenfelder gibt es wohl nicht wirklich.

  101. #101 Physik-Fan
    5. Oktober 2016

    @MartinB
    Ich habe doch die Gleichungen quasi explizit in dem 3. teil des oben verlinkten textes reingeschrieben!? Man muss wirklich nur g_munu als nen Feld betrachten, das an Energien koppelt; Energieterme in der QFT sind immer zweite Ableitungen nach der Zeit, fertig ist der Zeiteinfluss.

    Habe ich mir schon angesehen und der Grundgedanke, dass Grav. an Energie koppelt und dadurch an jedes Feld, hat was für sich. Aber für mich ist das von Dir Hingeschriebene zunächst rein formal. Es stellt sich die Frage, welche Physik steckt da drin? Was bedeutet das im Sinne von Wirkungsmechanismen?

    Eine spezielle Frage ist der Zusammenhang zwischen Energie und Zeit. Das hast Du ganz pauschal gesagt, für mich zu pauschal. Nehmen wir grav. Zeitdilatation, um die geht es ja hier. Sie tritt auch im statischen, homogenen Grav.feld auf, wo die Energiedichte überall gleich und konstant ist. Dann hängt die Zeitdilatation rein von der Höhe im Feld ab, also nicht von der Energiedichte.

    Naja, Einstein hatte für die ART zunächst auch keinerlei experimentelle Anhaltspunkte – die Theorie wurde rein deduktiv entwickelt, und erst dann hat Einstein mal nach Dingen wie Periheldrehung geguckt.

    Zwischen iterativ und deduktiv gibt es sicher eine Grauzone und auch die Meinungen, was man genau darunter versteht, gehen auseinander. In jeder Theorie steckt die Arbeit der Modellbildung. Wenn das bereits deduktiv wäre, wären Theorien immer deduktiv. Ich habe mich auf Randall bezogen und möchte auf das Kapitel 18 “Induktiv versus deduktiv” in “Die Vermessung des Universums” verweisen. Darin führt sie es genau aus. Interessanterweise spricht sie auch Einstein an, den jüngeren i.U. zum älteren. Beim Ersteren finden wir die RT, beim Letzteren die allg Feldtheorie. Die RT ist für sie induktiv, was ich auch so sehe. Du kennst ja die Entstehungsgeschichte der ART gut. Einstein hat sich bei der Konzeptentwicklung an der Empirie orientiert und sich mit Gedankenversuchen und Betrachtungen Stück für Stück die Physik erschlossen (freier Fall, Stab auf kreisförmiger Bahn, konvergierende Falllinien im Erdfeld etc.). Als er sich sicher war, dass das Konzept trägt, machte er sich an die Entwicklung der eigentl. Mathematik, mit Hilfe von Grossmann. Dabei war wieder die Empirie in Form überprüfbarer Effekte von zentraler Bedeutung. Schon früh, bevor die Feldgleichung in endgültiger Form vorlag, gewann er Freundlich für Messungen der Lichtablenkung in Sonnennähe (was letztendlich nicht von Erfolg gekrönt war). Mit der endgültigen Feldgleichung legte auch er die Lösung für diesen Fall vor. Der zweite Effekt war, wie von Dir genannt, die Periheldrehung des Merkur. Bei Entwicklung der ART war also immer der Bezug zur Empirie da.

    Übrigens, Randalls Vorgehensweise als rein induktiv anzusehen, wäre nicht richtig. Sie verwendet ja nicht beobachtbare Elemente, wie höhere Dimensionen mit Branen und Bulk. Sie bezeichnet sich als “Modellbauerin”, von Modellen aber, die sich experimentell überprüfen lassen. Das ist für sie entscheidend und unterscheidet sie von den String-Theoretikern.

  102. #102 MartinB
    12. Oktober 2016

    @Physik-Fan
    “Aber für mich ist das von Dir Hingeschriebene zunächst rein formal.”
    Das gilt umgekehrt genauso: ich kann mit der Feldformulierung starten und dann sagen “Die Interpretation als RZ-Krümmung ist rein formal”.

    Den zweiten Absatz habe ich nicht verstanden, worauf bezieht der sich genau?

    “Einstein hat sich bei der Konzeptentwicklung an der Empirie orientiert”
    Das ist jetzt eine Frage, was du “Empirie” nennst – eigentlich hat Einstein ja experimentelle Befunde (wie die Periheldrehung) zunächst gerade nicht verwendet (die erfolgreiche Rechnung dazu ist von 1915), sondern rein auf innerer Konsistenz und dem Äquivalenzprinzip beharrt.

  103. […] Bahn in der Raumzeit folgt – genau das ist ja gemeint, wenn man sagt, dass die Raumzeit gekrümmt ist. Im normalen Raum ist die geradeste Bahn diejenige, bei der der Abstand zwischen zwei Punkten auf […]